Tag Archives: 2019

Đề thi và lời giải Học sinh giỏi Quốc gia năm 2019 (VMO 2019)

Ngày thi thứ nhất. Thời gian làm bài 180 phút.

Bài 1. Cho hàm số liên tục $f: \mathbb{R} \rightarrow(0 ;+\infty)$ thỏa mãn

$\lim_{x \rightarrow – \infty} f(x)= \lim_{x \rightarrow + \infty} f(x) = 0$
a) Chứng minh rằng $f(x)$ đạt giá trị lớn nhất trên $\mathbb{R}$.
b) Chứng minh rằng tồn tại hai dãy $\left(x_n\right),\left(y_n\right)$ với $x_n<y_n, \forall n=1,2,3, \ldots$ sao cho chúng cùng hội tụ tới một giới hạn và thỏa mãn $f\left(x_n\right)=f\left(y_n\right)$ với mọi $n$.

Bài 2. Cho dãy số nguyên dương $\left(x_n\right)$ thỏa mãn $0 \leq x_0<x_1 \leq 100$ và
$$
x_{n+2}=7 x_{n+1}-x_n+280, \quad \forall n \geq 0 .
$$
a) Chứng minh rằng nếu $x_0=2, x_1=3$ thì với mỗi số nguyên dương $n$, tổng các ước nguyên dương của $x_n x_{n+1}+x_{n+1} x_{n+2}+x_{n+2} x_{n+3}+2018$ thì chia hết cho 24 .
b) Tìm tất cả các cặp số $\left(x_0, x_1\right)$ để số $x_n x_{n+1}+2019$ là số chính phương với vô số số $n$.

Bài 3. Với mỗi đa thức $f(x)=a_0+a_1 x+\cdots+a_n x^n$, đặt
$$
\Gamma(f(x))=a_0^2+a_1^2+\cdots+a_m^2 .
$$

Cho đa thức $P(x)=(x+1)(x+2) \ldots(x+2020)$. Chứng minh rằng tồn tại ít nhất 2019 đa thức đôi một phân biệt $Q_k(x)$ với $1 \leq k \leq 2^{2019}$ với các hệ số dương thỏa mãn hai điều kiện sau:
i) $\operatorname{deg} Q_k(x)=2020$.
ii) $\Gamma\left(Q_k(x)^n\right)=\Gamma\left(P(x)^n\right)$ với mọi số nguyên dương $n$.

Bài 4. Cho tam giác $A B C$ có tâm đường tròn nội tiếp $I$ và trực tâm $H$. Trên các tia $A B, A C, B C, B A, C A, C B$ lần lượt lấy các điểm $A_1, A_2, B_1, B_1, C_1, C_2$ sao cho $A A_1=A A_2=B C$, $B B_1=B B_2=A C, C C_1=C C_2=A B$. Gọi $A^{\prime}, B^{\prime}, C^{\prime}$ lần lượt là giao điểm của các cặp đường thẳng $\left(B B_1, C C_1\right) ;\left(C C_1, A A_1\right) ;\left(A A_1, B B_1\right)$.
a) Chứng minh rằng diện tích tam giác $A^{\prime} B^{\prime} C^{\prime}$ không vượt quá diện tích tam giác $A B C$.
b) Gọi $J$ là tâm đường tròn ngoại tiếp $A^{\prime} B^{\prime} C^{\prime}$. Các đường thẳng $A J, B J, C J$ lần lượt cắt $B C, C A, A B$ theo thứ tự tại $R, S, T$. Gọi $K$ là điểm chung của các đường tròn ngoại tiếp $A S T, B T R, C R S$. Giả sử tam giác $A B C$ không cân, chứng minh $I H J K$ là hình bình hành.

Ngày thi thứ hai. Thời gian làm bài 180 phút.

Bài 5. Xét đa thức $f(x)=x^2-\alpha x+1$ với $\alpha \in \mathbb{R}$.
a) Khi $\alpha=\frac{\sqrt{15}}{2}$, hãy viết $f(x)$ thành thương của hai đa thức với các hệ số không âm.
b) Tìm tất cả các giá trị $\alpha$ để $f(x)$ có thể viết được thành thương của hai đa thức với các hệ số không âm.

Bài 6. Cho tam giác nhọn, không cân $A B C$ nội tiếp đường tròn $(O)$ và có trực tâm $H$. Gọi $M, N, P$ lần lượt là trung điểm cạnh $B C, C A, A B$ và $D, E, F$ lần lượt là chân đường cao ứng với các đỉnh $A, B, C$ của tam giác $A B C$. Gọi $K$ là đối xứng của $H$ qua $B C$. Hai đường thẳng $D E, M P$ cắt nhau tại $X$; hai đường thẳng $D F, M N$ cắt nhau tại $Y$.
a) Đường thẳng $X Y$ cắt cung $\overparen{B C}$ của $(O)$ tại $Z$. Chứng minh rằng $K, Z, E, F$ đồng viên.
b) Hai đường thẳng $K E, K F$ cắt lại $(O)$ tại $S, T$. Chứng minh rằng $B S, C T, X Y$ đồng quy.

Bài 7. Có một số mảnh giấy hình vuông có cùng kích thước, mỗi mảnh được chia caro thành $5 \times 5$ ô vuông ở cả hai mặt. Ta dùng $n$ màu để tô các mảnh giấy sao cho mỗi ô của mỗi mảnh giấy được tô cả hai mặt bởi cùng một màu. Hai mảnh giấy màu được coi là giống nhau nếu có thể xếp chúng chồng khít lên nhau sao cho các cặp ô vuông ở cùng vị trí có cùng màu. Chứng minh rằng ta thu được không quá $\frac{1}{8}\left(n^{25}+4 n^{15}+n^{13}+2 n^7\right)$ mảnh giấy đôi một không giống nhau.

Lời giải tham khảo

Đề thi và đáp án kì thi chọn đội tuyển thi Quốc gia trường Phổ thông Năng khiếu năm học 2019 – 2020

ĐỀ THI

Ngày thi thứ nhất

Bài 1. Số thực $\alpha$ được gọi là điểm tụ của dãy số $\left(u_n\right)$ nếu tồn tại ít nhất một dãy con của $\left(u_n\right)$ có hội tụ đến $\alpha$.

(a) Hãy chỉ ra một dãy số có vô hạn điểm tụ.

(b) Chứng minh rằng nếu dãy số có mọi dãy con hội tụ thì nó cũng hội tụ.

(c) Gọi $S$ là tập hợp tất cả các số chính phương dương. Dãy số $\left(a_n\right)$ xác định bởi $a_n=\frac{1}{n}$ nếu $n \in S$ và $a_n=\frac{1}{n^2}$ nếu $n \notin S$.

Đặt $b_n=\sum_{k=1}^n a_k$. Xét tính hội tụ của các dãy số $\left(a_n\right)$ và $\left(b_n\right)$.

Bài 2. Tìm tất cả các hợp số dương $n$ sao cho $\sigma(n) \equiv 2(\bmod \varphi(n))$, trong đó ký hiệu $\sigma(n), \varphi(n)$ là hàm tổng các ước của $n$ và hàm Euler.

Bài 3. Tìm tất cả các hàm số $f: \mathbb{R} \rightarrow \mathbb{R}$ thỏa mãn

$\quad\quad\quad\quad\quad f(f(x)+y)+f(x) f(f(y))=x f(y)+x+y, \forall x, y \in \mathbb{R} .$

Bài 4. Cho tam giác $A B C$ không cân nội tiếp trong đường tròn $(O)$ với $B C$ cố định và $A$ thay đổi trên cung lớn $B C$. Các đường tròn bàng tiếp góc $A, B, C$ lần lượt tiếp xúc với $B C, C A, A B$ tại $D, E, F$. Gọi $L, M, N$ lần lượt là giao điểm khác $A, B, C$ của $(A B E),(A C F) ;(B C F),(B A D) ;(C A D),(C B E)$.

(a) Chứng minh rằng $A L$ luôn đi qua điểm cố định khi $A$ thay đổi.

(b) Gọi $K, I, J$ lần lượt là trung điểm của $A D, B E, C F$. Chứng minh rằng $K L, I M, J N$ dồng quy.

Ngày thi thứ hai

Bài 5. Cho $a, b, c$ là các số thực dương thỏa mãn $8\left(a^2+b^2+c^2\right)=9(a b+b c+c a)$.

Tìm giá trị lớn nhất và giá trị nhỏ nhất của biểu thức

$\quad\quad\quad\quad\quad\quad\quad\quad\quad\quad T=\frac{a+b}{c}+\frac{b+c}{a}+\frac{c+a}{b} .$

Bài 6. Tìm tất cả các hàm số $f: \mathbb{Z}^{+} \rightarrow \mathbb{Z}^{+}$thỏa mãn đồng thời các điều kiện sau

$\quad\quad$ i) $m f(m)+n f(n)+2 m f(n)$ là số chính phương với mọi $m, n$;

$\quad\quad$ ii) $f(m n)=f(m) f(n)$ với mọi $m, n$ nguyên dương;

$\quad\quad$  iii) Với mọi số nguyên tố $p, f(p)$ không chia hết cho $p^2$.

Bài 7. Một trường phổ thông có $n$ học sinh. Các học sinh tham gia vào tổng cộng $m$ câu lạc bộ là $A_1, A_2, \ldots, A_m$.

(a) Chứng minh rằng nếu mỗi câu lạc bộ có 4 học sinh và hai học sinh bất kỳ tham gia chung nhất một câu lạc bộ thì $m \leq \frac{n(n-1)}{12}$.

(b) Giả sử tồn tại $k>0$ sao cho hai câu lạc bộ bất kỳ có chung nhau $k$ thành viên và tồn tại một câu lạc bộ $A_t$ có $k$ thành viên. Chứng minh rằng $m \leq n$

Bài 8. Cho tam giác $A B C$ nội tiếp đường tròn $(O)$. Đường tròn nội tiếp $(I)$ tiếp xúc với các cạnh $B C, C A, A B$ lần lượt tại $D, E, F$. Gọi $J$ là tâm bàng tiếp góc $A$ của tam giác $A B C$ và $H$ là hình chiếu của $D$ lên $E F$.

(a) Chứng minh rằng giao điểm của $A H, J D$ thì thuộc đường thẳng $O I$.

(b) Giả sử $D H$ cắt lại $(I)$ ở $K$ và $I K$ cắt lại đường tròn ngoại tiếp $(I E F)$ ở $L$. Chứng minh rằng $A D, L H$ cắt nhau tại một điểm nằm trên $(I E F)$.

 

LỜI GIẢI

Ngày thi thứ nhất

Bài 1. Số thực $\alpha$ được gọi là điểm tụ của dãy số $\left(u_n\right)$ nếu tồn tại ít nhất một dãy con của $\left(u_n\right)$ có hội tụ đến $\alpha$.

(a) Hãy chỉ ra một dãy số có vô hạn điểm tụ.

(b) Chứng minh rằng nếu một dãy số có mọi dãy con hội tụ thì nó cũng hội tụ.

(c) Gọi $S$ là tập hợp tất cả các số chính phương dương. Dãy số $\left(a_n\right)$ xác định bởi $a_n=\frac{1}{n}$ nếu $n \in S$ và $a_n=\frac{1}{n^2}$ nếu $n \notin S$.

Đặt $b_n=\sum_{k=1}^n a_k$. Xét tính hội tụ của các dãy số $\left(a_n\right)$ và $\left(b_n\right)$.

Lời giải. (a) Ta sẽ chỉ ra dãy số mà mỗi số nguyên dương xuất hiện vô hạn lần trong đó. Chẳng hạn $\left(u_n\right)$ : là

$\quad\quad\quad\quad\quad\quad\quad\quad\quad\quad 1,2,1,2,3,1,2,3,4,1,2,3,4,5, \ldots$

với $u_n=1$ nếu $n \in S$ và $u_{n+1}=u_n+1$ nếu $n \notin S$, trong đó $S$ là tập hợp các số có dạng $\frac{m(m+1)}{2}$ như $1,3,6,10,15, \ldots$ Khi đó, với mỗi số nguyên dương $m \in \mathbb{Z}^{+}$thì ta luôn có thể trích ra một dãy con vô hạn của $\left(u_n\right)$ có tất cả các phần tử đều bằng $m$, tức là hội tụ về $m$.

(b) Do mỗi dãy số là dãy con của chính nó nên rõ ràng khẳng định của bài toán là đúng.

(c) Ta có $0 \leq a_n \leq \frac{1}{n}$ với mọi $n$ nên theo nguyên lí kẹp, ta suy ra $\lim a_n=0$. Nhận xét rằng $b_n$ là dãy tăng. Ta có

$\quad\quad\quad\quad\quad b_{n^2}=\sum_{i=1}^{n^2} a_i =\sum_{i \in S, i=1}^{n^2} a_i+\sum_{i \notin S, i=1}^{n^2} a_i=\sum_{i \in S, i=1}^{n^2} \frac{1}{i}+\sum_{i \notin S, i=1}^{n^2} \frac{1}{i^2} $

$\quad\quad\quad\quad\quad\quad\quad\quad\quad\quad\quad =\left(1+\frac{1}{2^2}+\cdots+\frac{1}{n^2}\right)+\sum_{i \notin S, i=1}^{n^2} \frac{1}{i^2} $

$\quad\quad\quad\quad\quad\quad\quad\quad\quad\quad\quad <\left(1+\frac{1}{2^2}+\cdots+\frac{1}{n^2}\right)+\left(\sum_{i=1}^{n^2} \frac{1}{i^2}\right) $

$\quad\quad\quad\quad\quad\quad\quad\quad\quad\quad\quad =\left(1+\frac{1}{2^2}+\cdots+\frac{1}{n^2}\right)+\left(1+\frac{1}{2^2}+\cdots+\frac{1}{n^4}\right)$

Vì dãy $u_n=1+\frac{1}{2^2}+\cdots+\frac{1}{n^2}<1+\frac{1}{1 \cdot 2}+\frac{1}{2 \cdot 3}+\cdots+\frac{1}{(n-1) n}=2-\frac{1}{n}<2$ là bị chặn trên nên từ đánh giá đã xây dựng được, ta có $b_{n^2}$ cũng bị chặn trên. Kết hợp với $b_n$ là dãy tăng, ta suy ra bản thân dãy $b_n$ cũng bị chặn trên nên nó hội tụ.

Bài 2. Tìm tất cả các hợp số dương $n$ sao cho

$\quad\quad\quad\quad\quad\quad\quad\quad\quad\quad \sigma(n) \equiv 2 \quad(\bmod \varphi(n)),$

trong đó ký hiệu $\sigma(n), \varphi(n)$ là hàm tổng các ước của $n$ và hàm Euler.

Lời giải . Giả sử $p$ là một ước nguyên tố lẻ của $n$. Nếu $v_p(n)>1$ thì theo công thức của hàm Euler, ta có $p \mid \varphi(n)$, mà $n \cdot \sigma(n)-2$ chia hết cho $\varphi(n)$, tức là cũng chia hết cho $p$ nên kéo theo $p \mid 2$, vô lý. Suy ra $v_p(n)=1$ với mọi $p \mid n$.

Đặt $n=2^k \cdot p_1 p_2 \ldots p_t$ với $k \geq 0$ và $p_1<p_2<\ldots<p_t$ là các số nguyên tố phân biệt. Theo công thức tính các hàm, ta có

$\quad\quad\quad\quad\quad\quad\quad \varphi(n)=2^{k-1}\left(p_1-1\right)\left(p_2-1\right) \ldots\left(p_t-1\right)$

$\quad\quad\quad\quad\quad\quad\sigma(n)=\left(2^{k+1}-1\right)\left(p_1+1\right)\left(p_2+1\right) \ldots\left(p_t+1\right) .$

Đánh giá lũy thừa 2 trong các số trên, ta có

$\quad\quad\quad\quad\quad\quad v_2(\varphi(n)) \geq k-1+t \text { và } v_2(n \cdot \sigma(n)) \geq k+t .$

Do đó từ $\varphi(n) \mid n \cdot \sigma(n)-2$, ta suy ra $1 \geq k-1+t$ nên $k+t \leq 2$. Ta xét các trường hợp sau

  • Nếu $t=0$ thì $n=2^k$ là hợp số nên $k=2, n=4$, thử trực tiếp ta thấy thỏa.
  • Nếu $t=1$ thì $n=2 p$ nên $\varphi(n)=p-1, \sigma(n)=3(p+1)$ và đưa về

$\quad\quad\quad\quad\quad\quad\quad\quad\quad\quad p-1 \mid 6 p(p+1)-2$

Chú ý rằng

$\quad\quad\quad\quad\quad\quad 6 p(p+1)-2=6 p^2+6 p-2=(p-1)(6 p+12)-10$

nên $p-1 \mid 10$. Từ đó ta tìm được $p=3, p=11$ tương ứng với $n=6, n=22$.

  • Nếu $t=2$ thì $k=0$, ta có $n=p_1 p_2$ nên

$\quad\quad\quad\quad \varphi(n)=\left(p_1-1\right)\left(p_2-1\right) \text { và } \sigma(n)=\left(p_1+1\right)\left(p_2+1\right)$

đưa về

$\quad\quad\quad\quad \left(p_1-1\right)\left(p_2-1\right) \mid\left(p_1+1\right)\left(p_2+1\right)-2 .$

Điều này không thể xảy ra vì $\left(p_1-1\right)\left(p_2-1\right)$ chia hết cho 4 trong khi biểu thức còn lại chia 4 dư 2 . Do đó, trường hợp này không có số $n$ thỏa mãn.

Vậy tất cả các số cần tìm là $4,6,22$.

Nhận xét. Chú ý rằng mọi số nguyên tố đều thỏa mãn yêu cầu của đề bài.

Bài 3. Tìm tất cả các hàm số $f: \mathbb{R} \rightarrow \mathbb{R}$ thỏa mãn

$\quad\quad\quad\quad\quad\quad f(f(x)+y)+f(x) f(f(y))=x f(y)+x+y$

với mọi số thực $x, y$.

Lời giải. Thay $x=y=0$ vào phương trình đề cho, ta có

$\quad\quad\quad\quad\quad\quad\quad\quad\quad\quad f(f(0))+f(0) f(f(0))=0 .$

suy ra $f(f(0))=0$ hoặc $f(0)=-1$. Ta xét các trường hợp sau:

  1. Nếu $f(f(0))=0$. Thay $y=0$, vào phương trình dề cho, ta có $f(f(x))=$ $x f(0)+x, \forall x \in \mathbb{R}$ Thay $x=f(0)$ và sử dụng $f(f(0))=0$, ta được $f(0)=$ $[f(0)]^2+f(0)$, hay $f(0)=0$. Do đó $f(f(x))=x$ với mọi $x \in \mathbb{R}$. Thay vào phương trình đề bài, ta có

$\quad\quad\quad\quad\quad\quad f(f(x)+y)+y f(x)=x f(y)+x+y, \forall x, y \in \mathbb{R} .$

Thay $y$ bởi $f(y)$ và sử dụng tính đối xứng của vế trái, ta được

$\quad\quad\quad\quad\quad\quad f(f(x)+f(y))+f(x) f(y)=x y+x+f(y)=x y+y+f(x) .$

Do đó $f(x)-x=f(y)-y$ với mọi $x, y \in \mathbb{R}$, hay $f(x)=x+c$. Thử lại, ta có $c=0$.

  1. Nếu $f(0)=-1$. Thay $y=0$ vào phương trình đề cho, ta có $f(f(x))+$ $f(x) f(-1)=0, \forall x \in \mathbb{R}$. Từ đây suy ra $f(f(-1))=-[f(-1)]^2$. Thay $x=0$ vào phương trình đề cho, ta có $f(y-1)-f(f(y))=y, \forall y \in \mathbb{R}$. Kết hợp các đẳng thức trên lại, ta có

$\quad\quad\quad\quad\quad\quad\quad\quad f(x-1)+f(x) f(-1)=x, \forall x \in \mathbb{R} .$

Thay $y=-1$ vào phương trình đề cho và sử dụng $f(f(-1))=-[f(-1)]^2$, ta lại có

$\quad\quad\quad\quad\quad\quad f(f(x)-1)-f(x)[f(-1)]^2=x f(-1)+x-1, \forall x \in \mathbb{R} .$

Mặt khác, ta cũng có

$\quad\quad\quad\quad\quad\quad\quad\quad f(-1) f(f(x))+f(x)[f(-1)]^2=0, \forall x \in \mathbb{R} .$

Cộng vế theo vế hai biểu thức trên lại, ta có

$\quad\quad\quad\quad\quad\quad\quad\quad\quad\quad f(x)=[1+f(-1)] x+1, \forall x \in \mathbb{R} .$

Thử lại, ta thấy không thỏa mãn.

Vậy phương trình có nghiệm hàm duy nhất là $f(x)=x$.

Bài 4. Cho tam giác $A B C$ không cân nội tiếp trong đường tròn $(O)$ với $B C$ cố định và $A$ thay đổi trên cung lớn $B C$. Các đường tròn bàng tiếp góc $A, B, C$ lần lượt tiếp xúc với các cạnh $B C, C A, A B$ tại $D, E, F$. Gọi $L, M, N$ lần lượt là giao điểm khác $A, B, C$ của các cặp đường tròn

$\quad\quad\quad\quad (A B E),(A C F) ;(B C F),(B A D) ;(C A D),(C B E) .$

(a) Chứng minh rằng $A L$ luôn đi qua điểm cố định khi $A$ thay đổi.

(b) Gọi $K, I, J$ lần lượt là trung điểm của $A D, B E, C F$. Chứng minh rằng $K L, I M, J N$ đồng quy.

Lời giải . (a) Đặt $B C=a, C A=b, A B=c$ và $p$ là nửa chu vi thì theo tính chất tiếp điểm bàng tiếp, ta có $B F=C E=p-a$.

Bằng biến đổi góc, ta có được $\triangle L B F \sim \triangle L E C(g . g)$, mà $B F=C E$ nên hai tam giác này bằng nhau. Suy ra $L B=L E, L C=L F$ nên $L$ là trung điểm cung $B E$ của đường tròn $(A B E)$ và cũng là trung diểm cung $C F$ của $(A C F)$.

Từ đó ta có $A L$ là phân giác góc $B A C$ hay $A L$ luôn đi qua trung điểm cung nhỏ $B C$ của $(O)$, là điểm cố định.

(b) Để ý rằng vai trò của $M, N, L$ là bình đẳng trong tam giác $A B C$. Do đó, từ câu a, một cách tương tự, ta có $M, N$ thuộc phân giác góc $B, C$ nên cũng lần lượt là trung điểm các cung nhỏ của các đường tròn tương ứng. Suy ra $M, K, N$ thẳng hàng (cùng thuộc trung trực của đoạn $A D$ ); tương tự với các bộ ba $N, I, L$ và $L, J, M$. Cuối cùng, ta thấy rằng

$\quad\quad\quad\quad\quad\quad\quad \frac{K M}{K N}=\frac{A K \cdot \tan \angle M A K}{A K \cdot \tan \angle N A K}=\frac{\tan (B / 2)}{\tan (C / 2)} .$

Tương tự với các tỷ số khác. Đến đây, áp dụng định lý Ceva cho tam giác $M N L$, ta có các đoạn thẳng $L K, I M, J N$ dồng quy.

Nhận xét. Một cách khác cho câu a như sau: Xét phép nghịch đảo đối xứng với phương tích $k=A B \cdot A C$ và trục đối xứng là phân giác góc $A$. Ta có $E \rightarrow E^{\prime} \in$ $A C, F \rightarrow F^{\prime} \in A B$ sao cho $A E \cdot A E^{\prime}=A F \cdot A F^{\prime}=k$. Ta tính được

$\quad\quad\quad\quad A E^{\prime}=\frac{b c}{p-c} \rightarrow B E^{\prime}=\frac{c(p-a)}{p-c} \rightarrow \frac{E^{\prime} B}{E^{\prime} A}=\frac{p-a}{b} .$

Tương tự thì $\frac{F^{\prime} C}{F^{\prime} A}=\frac{p-a}{c}$. Áp dụng định lý Ceva cho tam giác $A B C$ thì $C E^{\prime}, B F^{\prime}$ và phân giác góc $A$ đồng quy.

Lại có qua phép nghịch đối xứng trên thì phân giác giữ nguyên,

$\quad\quad\quad\quad\quad\quad\quad\quad\quad\quad (A B E) \rightarrow C F^{\prime},(A C F) \rightarrow B E^{\prime}$

nên ta có $L$ thuộc phân giác góc $A$.

 

Ngày thi thứ hai

Bài 5. Cho $a, b, c$ là các số thực dương thỏa mãn $8\left(a^2+b^2+c^2\right)=9(a b+b c+c a)$. Tìm giá trị lớn nhất và giá trị nhỏ nhất của biểu thức

$\quad\quad\quad\quad\quad\quad\quad\quad\quad\quad T=\frac{a+b}{c}+\frac{b+c}{a}+\frac{c+a}{b} .$

Lời giải . Do tính thuần nhất đối xứng của các biến nên chuẩn hóa

$\quad\quad\quad\quad\quad a b+b c+c a=8 \rightarrow a^2+b^2+c^2=9 \rightarrow a+b+c=5 .$

Ta có $P+3=(a+b+c)\left(\frac{1}{a}+\frac{1}{b}+\frac{1}{c}\right)=\frac{40}{a b c}$ nên ta đưa về tìm min, max của $T=a b c$ trong điều kiện

$\quad\quad\quad\quad\quad\quad\quad\quad\quad\quad\quad \left\{\begin{array}{l}a+b+c=5 \\ a b+b c+c a=8\end{array}\right.$

Chú ý rằng $b+c=5-a, b c=8-a(b+c)=8-a(5-a)$ nên từ đánh giá quen thuộc $(b+c)^2 \geq 4 b c$, ta có

$\quad\quad\quad\quad\quad\quad (5-a)^2 \geq 4\left(8-5 a+a^2\right) \Leftrightarrow 1 \leq a \leq \frac{7}{3} .$

Suy ra $T=a b c=a\left(8+a^2-5 a\right)=f(a)$. Đến đây khảo sát hàm số này trên miền $\left[1 ; \frac{7}{3}\right]$, ta được $\min T=4, \max T=\frac{112}{27}$ nên $\min P=\frac{93}{14}$, $\max P=7$. Từ đó, ta thu được kết luận như sau

  • Giá trị lớn nhất của $P$ là 7 , đạt được chẳng hạn khi $(a, b, c)=(2,2,1)$.
  • Giá trị nhỏ nhất của $P$ là $\frac{93}{14}$, đạt được chẳng hạn khi $(a, b, c)=\left(\frac{7}{3}, \frac{4}{3}, \frac{4}{3}\right)$.

Bài 6. Tìm tất cả các hàm số $f: \mathbb{Z}^{+} \rightarrow \mathbb{Z}^{+}$thỏa mãn đồng thời các điều kiện sau đây

$\quad\quad$ i) $m f(m)+n f(n)+2 m f(n)$ là số chính phương với mọi $m, n$;

$\quad\quad$  ii) $f(m n)=f(m) f(n)$ với mọi $m, n$ nguyên dương;

$\quad\quad$  iii) Với mọi số nguyên tố $p, f(p)$ không chia hết cho $p^2$.

Lời giải . Thay $m=n=1$ vào ii), ta suy ra $f(1)=f(1)^2$ nên $f(1)=1$. Thay $m=n$ vào i), ta suy ra $4 m f(m)$ là số chính phương với mọi $m \in \mathbb{Z}^{+}$nên $m f(m)$ cũng là số chính phương với mọi $m \in \mathbb{Z}^{+}$.

Với $p$ là số nguyên tố, vì $p f(p)$ là số chính phương nên $p \mid f(p)$ và ta đặt $f(p)=k^2 p$, với $k$ là số nguyên dương nào đó. Thay $m=p, n=1$ vào i), ta suy ra $p f(p)+1+2 p$ là số chính phương, hay $k^2 p^2+2 p+1$ là số chính phương.

Vì $k^2 p^2+2 p+1>(k p)^2$ nên

$\quad\quad\quad\quad\quad\quad k^2 p^2+2 p+1 \geq(k p+1)^2=k^2 p^2+2 k p+1 .$

Do đó $2 p \geq 2 k p$ nên ta phải có $k=1$.

Vì thế nên $f(p)=p$ với mọi số nguyên tố $p$. Sử dụng điều kiện ii), hàm $f$ nhân tính, và cũng vì mọi số nguyên dương bất kỳ đều có thể viết dưới dạng tích của các số nguyên tố nên ta có được $f(n)=n$ với mọi $n \in \mathbb{Z}^{+}$.

Thử lại ta thấy hàm số này thỏa mãn các ràng buộc của đề bài.

Nhận xét. Trên thực tế, ta có thể bỏ bớt diều kiện ii), iii) đi mà bài toán gốc vẫn có thể giải quyết được. Cụ thể như sau:

Chứng minh rằng nếu hàm số $f: \mathbb{Z}^{+} \rightarrow \mathbb{Z}^{+}$thỏa mãn $f(1)=1$ và với mọi $m, n \in \mathbb{Z}^{+}$, ta có $m f(m)+n f(n)+2 m f(n)$ là số chính phương thì $f(n)=n, \forall n \in \mathbb{Z}^{+}$.

Bài 7. Một trường phổ thông có $n$ học sinh. Các học sinh tham gia vào tổng cộng $m$ câu lạc bộ là $A_1, A_2, \ldots, A_m$.

(a) Chứng minh rằng nếu mỗi câu lạc bộ có 4 học sinh và hai học sinh bất kỳ tham gia chung nhất một câu lạc bộ thì $m \leq \frac{n(n-1)}{12}$.

(b) Giả sử tồn tại $k>0$ sao cho hai câu lạc bộ bất kỳ có chung nhau $k$ thành viên và tồn tại một câu lạc bộ $A_t$ có $k$ thành viên. Chứng minh rằng $m \leq n$

Lời giải . (a) Gọi $S$ là số bộ $({A, B}, C)$ mà trong đó học sinh $A, B$ cùng tham gia vào câu lạc bộ $C$. Ta thực đếm $S$ bằng hai cách

  1. Chọn câu lạc bộ trước, có $m$ cách, chọn cặp học sinh cùng tham gia vào đó có $C_4^2=6$ cách nên $S=6 \mathrm{~m}$.
  2. Chọn cặp học sinh trước, có $C_n^2$ cách, chọn câu lạc bộ mà hai học sinh đó cùng tham gia, có không quá 1 cách nên $S \leq C_n^2$.

Từ đó suy ra

$\quad\quad\quad\quad\quad\quad\quad\quad\quad 6 m \leq C_n^2 \Leftrightarrow m \leq \frac{n(n-1)}{12} .$

(b) Xét câu lạc bộ $X$ nào đó có $k$ thành viên. Xét $m-1$ câu lạc bộ còn lại thì theo giả thiết, rõ ràng các câu lạc bộ này đều có chứa $k$ thành viên trên của câu lạc bộ $X$. Từ đó suy ra $m-1$ câu lạc bộ còn lại đôi một không có thành viên chung.

Xét $n-k$ học sinh còn lại của trường thì rõ ràng một học sinh thuộc tối đa một câu lạc bộ (trong số các câu lạc bộ còn lại), suy ra số câu lạc bộ còn lại không vượt quá $n-k$ nên suy ra $m \leq n-k+1 \leq n$. Ta có điều phải chứng minh.

Nhận xét. Ý b của bài toán khá hiển nhiên, nhưng thực ra nó là một “phiên bản dễ” của bất đẳng thức Fisher sau đây:

Cho $A_1, A_2, \ldots, A_m$ là các tập con của tập ${1,2, \ldots, n}$ sao cho hai tập con bất kỳ có chung nhau đúng $k$ (với $k$ là số nguyên cố định nào đó không vượt quá n). Khi đó $m \leq n$.

Tuy nhiên, chứng minh sơ cấp cho kết quả này quả thực rất khó. Cách phổ biến nhất là dùng đại số tuyến tính. Cụ thể là:

Ta đặt tương ứng mỗi tập $A_i$ với một vector $v_i$ trong $\mathbb{F}_2^n$ như sau

$\quad\quad\quad\quad\quad\quad\quad\quad v_{i j}=\left\{\begin{array}{l}1 \text { nếu } j \in A_i \\ 0 \text { nếu } j \notin A_i\end{array} .\right.$

Chú ý rằng $\left|A_i \cap A_j\right|=k$ với mọi $i \neq j$. Bởi vậy, các vector $v_1, \ldots, v_m$ là các vector trong $\mathbb{R}^n$. Mặt khác, ta có số chiều của $\mathbb{R}^n$ là $n$. Do đó, trong bước tiếp theo chúng ta chỉ cần chứng minh $v_1, \ldots, v_m$ độc lập tuyến tính trong không gian $\mathbb{R}^n$.

Giả sử phản chứng rằng tồn tại các hệ số $\alpha_1, \ldots \alpha_m$ không đồng nhất bằng không sao cho $\sum_{i=1}^m \alpha_i v_i=0$. Do đó, ta có

$\quad\quad\quad\quad\quad\quad 0 =\left|\sum_{i=1}^m \alpha_i v_i\right|^2=\left\langle\sum_{i=1}^m \alpha_i v_i, \sum_{i=1}^m \alpha_i v_i\right\rangle $

$\quad\quad\quad\quad\quad\quad\quad =\sum_{i=1}^m \alpha_i^2\left|v_i\right|^2+\sum_{1 \leq i \neq j \leq m} \alpha_i \alpha_j\left\langle v_i, v_j\right\rangle$

Mặt khác, $\left|v_i\right|^2=\left|A_i\right|$, và $\left\langle v_i, v_j\right\rangle=\left|A_i \cap A_j\right|$. Bởi vậy,

$\quad\quad\quad\quad 0=\sum_{i=1}^m \alpha_i^2\left|v_i\right|^2+\sum_{i \neq j} k \alpha_i \alpha_j=\sum_{i=1}^m \alpha_i^2\left(\left|A_i\right|-k\right)+k \sum_{1 \leq i, j \leq m} \alpha_i \alpha_j$

Ta thấy rằng $\sum_{1 \leq i, j \leq m} \alpha_i \alpha_j=\left(\sum_{1 \leq i \leq m} \alpha_i\right)^2$, nên $0=\sum_{i=1}^m \alpha_i^2\left(\left|A_i\right|-k\right)+k\left(\sum_{1 \leq i, j \leq m} \alpha_i\right)^2$. Vì $\left|A_i\right| \geq k$ và có nhiều nhất một tập với chính xác $k$ phần tử, nên $\alpha_1=\cdots=\alpha_m=0$.

Điều này mâu thuẫn với giả thiết, hay các vector $v_1, \ldots, v_m$ là độc lập tuyến tính. Như vậy ta sẽ có $m \leq n$.

Bài 8. Cho tam giác $A B C$ nội tiếp đường tròn $(O)$. Đường tròn nội tiếp $(I)$ tiếp xúc với các cạnh $B C, C A, A B$ lần lượt tại $D, E, F$. Gọi $J$ là tâm bàng tiếp góc $A$ của tam giác $A B C$ và $H$ là hình chiếu của $D$ lên $E F$.

(a) Chứng minh rằng giao điểm của $A H, J D$ thì thuộc đường thẳng $O I$.

(b) Giả sử $D H$ cắt lại $(I)$ ở $K$ và $I K$ cắt lại đường tròn ngoại tiếp $(I E F)$ ơ $L$. Chứng minh rằng $A D, L H$ cắt nhau tại một diểm nằm trên $(I E F)$.

Lời giải. (a) Ta có bổ đề sau:

Bổ ĐỀ. $O I$ là đường thẳng Euler của tam giác $D E F$.

Bổ đề này quen thuộc và có thể chứng minh bằng cách hướng như sau (chi tiết xin dành cho bạn đọc).

  1. Sử dụng phép nghịch đảo tâm $I$, phương tích $r^2$ biến $(O)$ thành đường tròn Euler của $D E F$ nên có các tâm thẳng hàng.
  2. Sử dụng phép vị tự bằng cách gọi thêm trung điểm các cung nhỏ $B C, C A, A B$ của $(O)$.

Khi đó, gọi $T$ là giao điểm của $I O$ và $H D$ thì rõ ràng $T$ là trực tâm của tam giác $D E F$. Gọi $M$ là trung điểm cung nhỏ $B C$ của $(O)$ thì dễ thấy $M$ là trung điểm $I J$.

Bằng biến đổi góc, ta có $\triangle T E F \sim \triangle I B C$, mà $T H, I D$ là hai đường cao tương ứng nên $\frac{T H}{I D}=\frac{E F}{B C}$. Mặt khác, $\triangle I E F \sim \triangle M B C$ nên

$\quad\quad\quad\quad\quad\quad\quad\quad \frac{E F}{B C}=\frac{I E}{M C}=\frac{2 I E}{2 M I}=\frac{2 I E}{I J}$

suy ra $\frac{T H}{I D}=\frac{2 I E}{I J}$. Do đó

$\quad\quad\quad\quad\quad\quad T H \cdot I J=2 I D^2=2 I N \cdot I A=T D \cdot I A$

(vì $I, T$ lần lượt là tâm ngoại tiếp và trực tâm tam giác $D E F$ ) nên $\frac{T H}{T D}=\frac{I A}{I J}$. Cuối cùng, vì $H D | A J$ (cùng vuông góc với $E F$ ) nên theo định lý Talet thì $A H, J D, T I$ đồng quy hay nói cách khác, $A H, J D$ cắt nhau trên $O I$.

(b) Giả sử $A D$ cắt lại $(I)$ tại $G$. Ta cần chứng minh rằng $G, H, L$ thẳng hàng.

Do $D K | A I$ nên

$\quad\quad\quad\quad\quad\quad\quad\quad\quad\quad\angle A G L=\angle A I L=\angle A I K=\angle D K I$

suy ra $\angle D G L=\angle D K L$. Vì thế nên $D G K L$ là tứ giác nội tiếp. Do đó, $L G$ là trục đẳng phương của $(L K D),(I E F)$. Lại có

nên suy ra $H$ thuộc trục đẳng phương của hai đường tròn này, tức là $H \in L G$. Từ đó ta có điều phải chứng minh.

Nhận xét. Liên quan đến ý b, có một bài toán khá thú vị với nội dung như sau:

Trung tuyến đỉnh $D$ của tam giác $D E F$ cắt $(I)$ ở $L$. Chứng minh rằng trục đẳng phương của $(L B F),(L C E)$ đi qua giao điểm của $J D$ và đường thẳng qua $A$, vuông góc với $A I$.

 

 

 

 

 

 

 

 

 

 

 

 

 

 

 

 

 

 

 

 

 

 

 

 

 

 

 

 

 

 

 

 

 

 

 

 

 

 

 

 

 

 

 

 

 

 

 

 

 

 

 

 

 

 

 

 

 

 

 

 

 

 

 

 

 

 

 

 

 

 

 

 

 

 

 

 

 

 

 

 

 

 

 

 

 

 

 

 

 

 

 

 

 

 

 

 

 

 

 

 

Đề thi và đáp án kì thi chọn đội tuyển thi Quốc gia trường Phổ thông Năng khiếu năm học 2020 – 2021

ĐỀ THI

Ngày thi thứ nhất

Bài 1. Với mỗi số nguyên dương $n$, tìm số thực $M_n$ lớn nhất sao cho với mọi số thực dương $x_1, x_2, \ldots, x_n$ thì ta đều có

$\quad\quad\quad\quad\quad\quad\quad\quad\sum_{k=1}^n \frac{1}{x_k^2}+\frac{1}{\left(\sum_{k=1}^n x_k\right)^2} \geq M_n\left(\sum_{k=1}^n \frac{1}{x_k}+\frac{1}{\sum_{k=1}^n x_k}\right)^2$

Bài 2. Cho 2021 số nguyên khác 0 . Biết rằng tổng của một số bất kỳ trong chúng với tích của tất cả 2020 số còn lại luôn âm.

(a) Chứng minh rằng với mọi cách chia 2021 số này thành hai nhóm và nhân các số cùng nhóm lại với nhau thì tổng của hai tích cũng luôn âm.

(b) Một bộ số thỏa mãn đề bài thì có thể có nhiều nhất mấy số âm?

Bài 3. Cho hai hàm số $f: \mathbb{R} \rightarrow \mathbb{R}$ và $g: \mathbb{R} \rightarrow \mathbb{R}$ thỏa mãn $g(2020)>0$ và với mọi

$\quad\quad\quad\quad x, y \in \mathbb{R}$ thì $\left\{\begin{array}{l}f(x-g(y))=f(-x+2 g(y))+x g(y)-6 \\ g(y)=g(2 f(x)-y)\end{array}\right.$.

(a) Chứng minh rằng $g$ là hàm hằng.

(b) Chứng minh rằng đồ thị $h(x)=f(x)-x$ nhận $x=1$ là trục đối xứng.

Bài 4. Cho tam giác $A B C$ nhọn, nội tiếp trong đường tròn $(O)$ có trực tâm $H$ và $A H, B H, C H$ cắt cạnh đối diện lần lượt tại $D, E, F$. Gọi $I, M, N$ lần lượt là trung diểm các cạnh $B C, H B, H C$ và $B H, C H$ cắt lại $(O)$ theo thứ tự tại các điểm $L, K$. Giả sử $K L$ cắt $M N$ ở $G$.

(a) Trên $E F$, lấy diểm $T$ sao cho $A T$ vuông góc với $H I$. Chứng minh rằng $G T$ vuông góc với $O H$.

(b) Gọi $P, Q$ lần lượt là giao điểm của $D E, D F$ và $M N$. Gọi $S$ là giao điểm của $B Q, C P$. Chứng minh rằng $H S$ đi qua trung diểm của $E F$.

Ngày thi thứ hai

Bài 5. Cho số nguyên dương $n>1$. Chứng minh rằng với mọi số thực $a \in\left(0 ; \frac{1}{n}\right)$ và mọi đa thức $P(x)$ có bậc $2 n-1$ thỏa mãn điều kiện $P(0)=P(1)=0$, luôn tồn tại các số thực $x_1, x_2$ thuộc $[0 ; 1]$ sao cho $P\left(x_1\right)=P\left(x_2\right)$ và $x_2-x_1=a$.

Bài 6. Giải phương trình sau trên $\mathbb{Z}^{+}:\left(x^2+3\right)^{3^{x+1}}\left[\left(x^2+3\right)^{3^{x+1}}+1\right]+x^2+y=x^2 y$.

Bài 7. Cho các số nguyên $n>k>t>0$ và $X={1,2, \ldots, n}$. Gọi $\mathcal{F}$ là họ các tập con có $k$ phần tử của tập hợp $X$ sao cho với mọi $F, F^{\prime} \in \mathcal{F}$ thì $\left|F \cap F^{\prime}\right| \geq t$. Giả sử không có tập con có $t$ phần tử nào chứa trong tất cả các tập $F \in \mathcal{F}$.

(a) Chứng minh rằng tồn tại một tập hợp $B \subset X$ sao cho $|B|<3 k$ và $|B \cap F| \geq t+1$ với mọi $F \in \mathcal{F}$.

(b) Chứng minh rằng $|\mathcal{F}|<C_{3 k}^{t+1} C_n^{k-t-1}$.

Bài 8. Cho tam giác $A B C$ nội tiếp trong $(O)$ với $B, C$ cố định và $A$ thay đổi trên cung lớn $B C$. Dựng hình bình hành $A B D C$ và $A D$ cắt lại $(B C D)$ ở $K$.

(a) Gọi $R_1, R_2$ lần lượt là bán kính đường tròn ngoại tiếp $(K A B),(K A C)$. Chứng minh rằng tích $R_1 R_2$ không đổi.

(b) Ký hiệu $(T),\left(T^{\prime}\right)$ lần lượt là các đường tròn cùng đi qua $K$, tiếp xúc với $B D$ ở $B$ và tiếp xúc với $C D$ ở $C$. Giả sử $(T),\left(T^{\prime}\right)$ cắt nhau ở $L \neq K$. Chứng minh rằng $A L$ luôn đi qua một điểm cố định.

LỜI GIẢI

Ngày thi thứ nhất

Bài 1. Với mỗi số nguyên dương $n$, tìm số thực $M_n$ lớn nhất sao cho với mọi số thực dương $x_1, x_2, \ldots, x_n$ thì ta đều có

$\quad\quad\quad\quad\quad\quad\quad\quad\sum_{k=1}^n \frac{1}{x_k^2}+\frac{1}{\left(\sum_{k=1}^n x_k\right)^2} \geq M_n\left(\sum_{k=1}^n \frac{1}{x_k}+\frac{1}{\sum_{k=1}^n x_k}\right)^2 .$

Lời giải.

Điều Kiện Cần. Với $x_1=x_2=\cdots=x_n=1$, ta có

$\quad\quad\quad\quad\quad\quad\quad\quad n+\frac{1}{n^2} \geq M_n\left(n+\frac{1}{n}\right)^2 \text { hay } M_n \leq \frac{n^3+1}{\left(n^2+1\right)^2} .$

ĐIỀU KIỆN ĐỦ. Ta sẽ chứng minh hằng số trên thỏa mãn đề bài, tức là

$\quad\quad\quad\quad\quad\quad\quad\quad\sum_{k=1}^n \frac{1}{x_k^2}+\frac{1}{\left(\sum_{k=1}^n x_k\right)^2} \geq \frac{n^3+1}{\left(n^2+1\right)^2}\left(\sum_{k=1}^n \frac{1}{x_k}+\frac{1}{\sum_{k=1}^n x_k}\right)^2\quad\quad\quad\quad ( * )$

Theo bất đẳng thức Cauchy-Schwarz thì

$\quad\quad\quad\quad\quad\quad\quad\quad \sum_{k=1}^n \frac{1}{x_k^2} \geq \frac{1}{n}\left(\sum_{k=1}^n \frac{1}{x_k}\right)^2 .$

Đến đây, đặt $a=\sum_{k=1}^n \frac{1}{x_k}, b=\frac{1}{\sum_{k=1}^n x_k}$ là các số dương, ta đưa về chứng minh

$\quad\quad\quad\quad\quad\quad\quad\quad \frac{a^2}{n}+b^2 \geq \frac{n^3+1}{\left(n^2+1\right)^2}(a+b)^2 .$

Chú ý rằng $a \geq n^2 b$ nên quy đồng và khai triển, ta có

$\quad\quad\quad\quad\quad\quad\quad\quad \left(a-n^2 b\right)\left[\left(2 n^2-n+1\right) a-\left(n^3-n^2+2 n\right) b\right] \geq 0 .$

Chú ý rằng $n^2\left(2 n^2-n+1\right)-\left(n^3-n^2+2 n\right)=2\left(n^2+1\right) n(n-1) \geq 0$ nên

$\quad\quad\quad\quad\quad\quad\quad\quad \left(2 n^2-n+1\right) a \geq n^2\left(2 n^2-n+1\right) b \geq\left(n^3-n^2+2 n\right) b .$

Bất đẳng thức cuối đúng nên $(*)$ đúng. Vậy hằng số tốt nhất cần tìm là $M_{\max }=$ $\frac{n^3+1}{\left(n^2+1\right)^2}$

Nhận xét. Bài này là phiên bản tổng quát của câu bất đẳng thức trong đề Iran 2010 là

$\quad\quad\quad\quad\frac{1}{a^2}+\frac{1}{b^2}+\frac{1}{c^2}+\frac{1}{(a+b+c)^2} \geq \frac{7}{25}\left(\frac{1}{a}+\frac{1}{b}+\frac{1}{c}+\frac{1}{a+b+c}\right)^2 .$

Thực ra cách chứng minh cho trường hợp $n=3$ này cũng phản ánh cách xử lý cho trường hợp tổng quát trong bài toán ban đầu.

Bài 2. Cho 2021 số nguyên khác 0. Biết rằng tổng của một số bất kỳ trong chúng với tích của tất cả 2020 số còn lại luôn âm.

(a) Chứng minh rằng với mọi cách chia 2021 số này thành hai nhóm và nhân các số cùng nhóm lại với nhau thì tổng của hai tích cũng luôn âm.

(b) Một bộ số thỏa mãn đề bài thì có thể có nhiều nhất mấy số âm?

Lời giải: Đặt các số đã cho là $a_1, a_2, \ldots, a_{2021}$ và $S$ là tích của tất cả số này. Theo dề bài thì

$\quad\quad\quad\quad\quad\quad\quad\quad\quad\quad a_k+\frac{S}{a_k}=\frac{a_k^2+S}{a_k}<0$

với mọi $k=1,2, \ldots, 2021$.

Nếu như $S>0$ thì rõ ràng theo trên, ta phải có $a_k<0, \forall k$, nhưng điều này vô lý vì khi đó $S$ là tích của 2021 số âm nên cũng âm. Do đó, $S<0$ và trong các số đã cho, có lẻ số âm.

Nếu như số lượng số âm là lớn hơn 1 , giả sử hai trong các số đó là $a_1, a_2$ và $\left|a_2\right| \geq\left|a_1\right|$. Khi đó, ta có $a_2 a_3 \ldots a_{2021}>0$ (do trong các số này có chẵn số âm) và $\left|a_2 a_3 \ldots a_{2021}\right| \geq\left|a_1\right|$ nên

$\quad\quad\quad\quad\quad\quad\quad\quad\quad\quad a_1+a_2 a_3 \ldots a_{2021} \geq 0 .$

Điều vô lý này cho thấy không thể có nhiều hơn 1 số âm, và vì thế nên có đúng một số âm. Giả sử $a_1$ là số âm duy nhất đó thì ta có $a_1+a_2 a_3 \ldots a_{2021}<0$ nên

$\quad\quad\quad\quad\quad\quad\quad\quad\quad\quad \left|a_1\right|>a_2 a_3 \ldots a_{2021} .$

Với mọi cách chia 2021 số đã cho thành hai nhóm thì sẽ có một nhóm chứa số âm là $a_1$, đồng thời, giá trị tuyệt đối của tích các số trong nhóm đó, vì có chứa $\left|a_1\right|$, nên sẽ lớn hơn tích các số của nhóm còn lại. Suy ra tổng của hai tích sẽ âm.

Do đó, ta có khẳng định ở câu (a) và đáp số cho câu (b) là 1 .

Nhận xét. Nhờ việc khảo sát số lượng số âm, ta chỉ ra trực tiếp được rằng có đúng 1 số âm và các câu (a), (b) trở thành hiển nhiên. Bài toán cũng đúng khi thay 2021 số nguyên bởi $2 n+1$ số thực nào đó có giá trị tuyệt đối không nhỏ hơn 1 .

Bài 3. Cho hai hàm số $f: \mathbb{R} \rightarrow \mathbb{R}$ và $g: \mathbb{R} \rightarrow \mathbb{R}$ thỏa mãn $g(2020)>0$ và

$\quad\quad\quad\quad\quad\quad\quad\quad \left\{\begin{array}{l}f(x-g(y))=f(-x+2 g(y))+x g(y)-6 \\ g(y)=g(2 f(x)-y)\end{array}\right.$

với mọi $x, y \in \mathbb{R}$.

(a) Chứng minh rằng $g$ là hàm hằng.

(b) Chứng minh rằng đồ thị hàm số $h(x)=f(x)-x$ nhận $x=1$ là trục đối xứng.

Lời giải . (a) Trong điều kiện thứ nhất, thay $x$ bởi $\frac{3}{2} g(y)$ thì ta có ngay

$\quad\quad\quad\quad\quad\quad f\left(\frac{g(y)}{2}\right)=f\left(\frac{g(y)}{2}\right)+\frac{3}{2} g^2(y)-6 \text { hay } g^2(y)=4$

Suy ra $g(y)=\pm 2$ với mọi $y$. Do $g(2020)>0$ nên loại trường hợp $g(y)=-2, \forall y$. Ta sẽ chỉ ra rằng không xảy ra trường hợp $\exists a \neq b$ sao cho $g(a)=2, g(b)=-2$. Thay $y=2020$ vào điều kiện đầu, ta có

$\quad\quad\quad\quad\quad\quad\quad\quad\quad\quad f(x-c)-f(-x+2 c)=x c-6$

với $c=g(2020)>0$. Suy ra $f(u)-f(v)$ toàn ánh trên $\mathbb{R}$ với $u, v \in \mathbb{R}$. Tiếp theo trong diều kiện sau, thay $y$ bởi $a, b$, ta có

$\quad\quad\quad\quad\quad\quad g(a)=g(2 f(x)-a)=2 ; g(b)=g(2 f(x)-b)=-2$

với mọi $x \in \mathbb{R}$. Lại do tính toàn ánh, chọn $u, v$ sao cho

$\quad\quad\quad\quad\quad\quad\quad\quad\quad\quad f(u)-f(v)=\frac{a-b}{2}$

thì $2 f(u)-a=2 f(v)-b$. Suy ra

$\quad\quad\quad\quad\quad\quad\quad 2=g(2 f(u)-a)=g(2 f(v)-b)=-2,$

điều này vô lý. Vì thế $g(x)=2, \forall x$ nên $g$ là hàm hằng.

(b) Với $g(x)=2, \forall x$, thay vào điều kiện đầu, ta có

$\quad\quad\quad\quad\quad\quad\quad\quad f(x-2)=f(-x+4)+2 x-6$

hay

$\quad\quad\quad\quad f(x)=f(2-x)+2 x-2 \text { kéo theo } f(x)-x=f(2-x)-(2-x) .$

Điều này cho thấy $h(x)=h(2-x)$ hay $h(x)$ có đồ thị nhận $x=1$ là trục đối xứng.

Bài 4. Cho tam giác $A B C$ nhọn, nội tiếp trong đường tròn $(O)$ có trực tâm $H$ và $\mathrm{AH}, \mathrm{BH}, \mathrm{CH}$ cắt cạnh đối diện lần lượt tại $D, E, F$. Gọi $I, M, N$ lần lượt là trung điểm các cạnh $B C, H B, H C$ và $B H, C H$ cắt lại $(O)$ theo thứ tự tại các điểm $L, K$. Giả sử $K L$ cắt $M N$ ở $G$.

(a) Trên $E F$, lấy điểm $T$ sao cho $A T$ vuông góc với $H I$. Chứng minh rằng $G T$ vuông góc với $O H$.

(b) Gọi $P, Q$ lần lượt là giao điểm của $D E, D F$ và $M N$. Gọi $S$ là giao điểm của $B Q, C P$. Chứng minh rằng $H S$ di qua trung điểm của $E F$.

Lời giải. (a) Giả sử tia $I H$ cắt $(O)$ ở $R$ thì theo kết quả quen thuộc, ta có $\angle A R H=$ $90^{\circ}$. Vì thế nên $T \in A R$. Bằng cách xét trục đẳng phương của các đường tròn đường kính $A H, B C$ và đường tròn $(O)$, ta có $A R, E F, B C$ dồng quy. Từ đó suy ra $T \in B C$.

Gọi $\left(O^{\prime}\right)$ là đường tròn Euler của tam giác $A B C$ thì $D, E, F, M, N \in(O)$. Dễ thấy rằng

$\quad\quad\quad\quad H M \cdot H K=\frac{1}{2} H B \cdot H K=\frac{1}{2} H C \cdot H L=H N \cdot H L$

nên $M, N, K, L$ cùng thuộc đường tròn. Suy ra

$G L \cdot G K=G M \cdot G N$ nên

Ngoài ra, ta cũng có nên $G T$ chính là trục đăng phương của $(O),\left(O^{\prime}\right)$. Điều này cho thấy $G T \perp O O^{\prime}$ hay $G T \perp O H$ (do $O^{\prime}$ là trung diểm của $\left.O H\right)$.

(b) Ta có $D H$ là phân giác của góc $P D Q$, và $P Q \perp H D$ nên dễ thấy tứ giác $H P D Q$ là hình thoi. Ta biến đổi góc như sau

$\quad\quad\quad\quad\quad\quad\quad\quad \angle H P Q=\angle D Q P=\angle Q D B=\angle F H B .$

Suy ra $\angle H P N=\angle M H N$ nên $H N$ tiếp xúc với $(H M P)$ hay $N P \cdot N M=N H^2=$ $N C^2$. Do đó, hai tam giác $N P C$ và $N C M$ dồng dạng với nhau. Suy ra

$\quad\quad\quad\quad\quad\quad\quad\quad\quad\quad \angle N C P=\angle N M C=\angle M C B,$

nên $C P$ là dối trung của tam giác $H B C$. Chứng minh tương tự thì $B Q$ là dối trung trong tam giác $H B C$ nên điểm $S$ chính là điểm Lemoine của tam giác này, kéo theo $H S$ cũng là dối trung của tam giác $H B C$. Lại có $E F$ là dối song ứng với đỉnh $H$ trong tam giác $H B C$ nên suy ra $H S$ chia đôi đoạn thẳng $E F$.

 

Ngày thi thứ hai

Bài 5. Cho số nguyên dương $n>1$. Chứng minh rằng với mọi số thực $a \in$ $\left(0 ; \frac{1}{n}\right)$ và mọi đa thức $P(x)$ có bậc $2 n-1$ thỏa mãn điều kiện $P(0)=P(1)=0$, luôn tồn tại các số thực $x_1, x_2$ thuộc $[0 ; 1]$ sao cho $P\left(x_1\right)=P\left(x_2\right)$ và $x_2-x_1=a$.

Lời giải. Không mất tính tổng quát, giả sử không tồn tại các số $x_1, x_2$ thỏa mãn đề bài. Khi đó, xét đa thức $Q(x)=P(x+a)-P(x)$ sẽ vô nghiệm trên $[0 ; 1-a]$. Suy ra $Q(x)$ sẽ không đổi dấu trên miền đó vì nếu không, dùng tính liên tục thì sẽ mâu thuẫn. Không mất tính tổng quát, giả sử $Q(x)>0, \forall x \in[0 ; 1-a]$. Nhận xét rằng $\frac{1}{a} \notin \mathbb{Z}^{+}$vì nếu không, đặt $\frac{1}{a}=m \in \mathbb{Z}^{+}$thì ta sẽ có $m a=1$ và

$\quad\quad\quad\quad Q(a)+Q(2 a)+\cdots+Q((m-1) a)=P(1)-P(0)=0$

trong khi các số hạng ở vế trái đều dương, vô lý. Tiếp theo, ta có

$\quad\quad\quad\quad\quad\quad Q(a)=P(a)-P(0)>0 \Rightarrow P(a)>0$

Chứng minh tương tự thì $P(k a)>0, \forall k=1,2, \ldots, n$. Mặt khác,

$\quad\quad\quad\quad Q(1-a)=P(1)-P(1-a)=-P(1-a)>0 \text { nên } P(1-a)<0 .$

Tương tự thì

$\quad\quad\quad\quad\quad\quad\quad\quad\quad\quad P(1-l a)<0, \forall l=1,2, \ldots, n .$

Rõ ràng với mỗi $k \in{1,2, \ldots, n}$, ta luôn chọn được số nguyên $l_k \in{1,2, \ldots, n}$ để $(k-1) a<1-l_k a<k a$. Thật vậy, đánh giá này tương đương với

$\quad\quad\quad\quad\quad\quad\quad\quad\quad\quad \frac{1}{a}-k<l_k<\frac{1}{a}+1-k .$

Mà $a \in\left(0 ; \frac{1}{n}\right)$ nên $\frac{1}{a}-k>0$ và $\frac{1}{a}-k \notin \mathbb{Z}$ nên khoảng trên phải chứa một số nguyên và gọi số đó là $l_k$. Theo trên thì

$\quad\quad\quad\quad\quad\quad P((k-1) a)>0, P\left(1-l_k a\right)<0, P(k a)>0$

nên trên khoảng $((k-1) a ; k a)$ thì đa thức $P(x)$ có 2 nghiệm thực phân biệt. Áp dụng điều này cho tất cả các khoảng $(0 ; a),(a ; 2 a), \ldots,((n-1) a ; n a)$ thì ta thấy $P(x)$ sẽ có $2 n$ nghiệm thực phân biệt; trong khi $P(x)$ chỉ có bậc $2 n-1$, vô lý.

Vì thế nên điều giả sử là sai và tồn tại hai số thực $x_1, x_2 \in[0 ; 1]$ thỏa mãn đề bài.

Nhận xét. Ta thấy rằng do $P(x)$ đã có sã̃n hai nghiệm là $x=0, x=1$ nên lập luận trên vẫn đúng khi xét bậc đa thức là $2 n+1$ (vì ta đã chỉ ra được đến $2 n+2$ nghiệm phân biệt). Một phiên bản tương tự khi xét hàm số liên tục $f(x)$ tùy ý là:

Cho số nguyên dương $n$, xét hàm số $f(x)$ liên tục tục trên $[0 ; n]$ sao cho $f(0)=f(n)$. Khi đó, với $k \in{1,2, \ldots, n-1}$, tồn tại $x_1, x_2 \in[0 ; n]$ mà

$\quad\quad\quad\quad\quad\quad\quad\quad\quad\quad x_2-x_1=k \text { và } f\left(x_1\right)=f\left(x_2\right) .$

Bài 6. Giải phương trình sau trên tập số nguyên dương

$\quad\quad\quad\quad\quad\left(x^2+3\right)^{3^{x+1}}\left[\left(x^2+3\right)^{3^{x+1}}+1\right]+x^2+y=x^2 y .$

Lời giải . Dễ thấy $x=1$ không thỏa nên ta xét $x>1$. Thử trực tiếp thấy $x=2$ thỏa mãn. Xét $x>2$, kéo theo $x^2-1>3$. Phương trình đã cho viết lại thành

$\quad\quad\quad\quad\left(x^2+3\right)^{3^{x+1}}\left[\left(x^2+3\right)^{3^{x+1}}+1\right]+1=(y-1)\left(x^2-1\right) $

$\quad\quad\quad\quad \Leftrightarrow x^2-1\mid\left(x^2+3\right)^{3^{x+1}}\left[\left(x^2+3\right)^{3^{x+1}}+1\right]+1 $

$\quad\quad\quad\quad \Leftrightarrow 4^{3^{x+1}}\left(4^{3^{x+1}}+1\right)+1 \equiv 0 \quad\left(\bmod x^2-1\right) .$

Đặt $a=4^{3^{x+1}}$ thì ta có $a^2+a+1 \equiv 0\left(\bmod x^2-1\right)$ nên $a^3 \equiv 1\left(\bmod x^2-1\right)$ hay

$\quad\quad\quad\quad\quad\quad\quad\quad\quad\quad 4^{3^{x+2}} \equiv 1 \quad\left(\bmod x^2-1\right)$

Lại chú ý rằng
$\quad\quad\quad\quad\quad\quad a=4^{3^{x+1}}=4^{3 \cdot 3^x}=\left(4^3\right)^{3^x} \equiv 1 \quad(\bmod 9)$
nên nếu đặt $M=a^2+a+1$ thì $M \equiv 3(\bmod 9)$. Điều này cho thấy $v_3(M)=1$ nên $v_3\left(x^2-1\right) \leq 1$. Mà $x^2-1>3$ và dễ thấy $x^2-1$ lẻ nên $x^2-1$ phải có một ước nguyên tố lẻ $p>3$.

Đặt $h=\operatorname{ord}_p(4)$ thì $h \mid p-1$ và $h \mid 3^{x+2}$. Suy ra $h=3^k$ với $0 \leq k \leq x+2$. Tuy nhiên, nếu $k \leq x+1$ thì $4^{3^k} \equiv 1(\bmod p)$ mà $4^{3^k}-1 \mid a-1$ nên $a \equiv 1(\bmod p)$. Điều này vô lý do sẽ kéo theo
$\quad\quad\quad\quad\quad\quad\quad\quad\quad\quad 0 \equiv a^2+a+1 \equiv 3 \quad(\bmod p) .$
Vì thế nên $k=x+2$ hay $\operatorname{ord}_p(4)=3^{x+2}$ nên $h=3^{x+2} \leq p-1 \leq x-1$. Tuy nhiên, đánh giá này là không thể xảy ra với mọi $x>2$. Vì thế nên nghiệm duy nhất của phương trình dã cho là

$\quad\quad\quad\quad\quad\quad\quad\quad (x ; y)=\left(2 ; \frac{7^{27}\left(7^{27}+1\right)+4}{3}\right)$.

Nhận xét. Các bài toán lũy thừa tầng trong số học thường gợi ý đến việc dùng cấp theo kiểu: với mọi $a, m, k$ nguyên dương lớn hơn 1 và $p$ là số nguyên tố thì $a^{m^k} \equiv 1$ $(\bmod p)$ và $h=\operatorname{ord}_p(a)$ thì $h \mid m^k$ nên có $h=m^t$ với $t \leq k$. Một kết quả có liên quan là: mọi ước nguyên tố $p$ của $2^{2^n}+1$ thì đều thỏa $2^{n+1} \mid p-1$.

Bài 7. Cho các số nguyên $n>k>t>0$ và $X={1,2, \ldots, n}$. Gọi $\mathcal{F}$ là họ các tập con có $k$ phần tử của tập hợp $X$ sao cho với mọi $F, F^{\prime} \in \mathcal{F}$ thì $\left|F \cap F^{\prime}\right| \geq t$. Giả sử không có tập con có $t$ phần tử nào được chứa trong tất cả các tập $F \in \mathcal{F}$.

(a) Chứng minh rằng tồn tại một tập hợp $B \subset X$ sao cho $|B|<3 k$ và $|B \cap F| \geq t+1$ với mọi $F \in \mathcal{F}$.

(b) Chứng minh rằng $|\mathcal{F}|<C_{3 k}^{t+1} C_n^{k-t-1}$.

Lời giải. (a) Theo giả thiết thì rõ ràng $|\mathcal{F}| \geq 3$, vì nếu không thì diều kiện ii) sẽ không được thỏa mãn. Ta xét các trường hợp sau Nếu như mọi tập $F, F^{\prime} \in \mathcal{F}$ đều có tính chất $\left|F \cap F^{\prime}\right| \geq t+1$ thì ta chỉ cần chọn $B$ là một tập bất kỳ trong $\mathcal{F}$ là được, rõ ràng $|B|=k<3 k$, thỏa mãn đề bài.

Ngược lại, tồn tại hai tập $F, F^{\prime} \in \mathcal{F}$ mà $\left|F \cap F^{\prime}\right|=t$ thì theo giả thiết, không có tập con $t$ phần tử nào được chứa trong tất cả tập hợp của họ $\mathcal{F}$; vì thế nên phải có $F^{\prime \prime}$ sao cho $\left(F \cap F^{\prime}\right) \not \subset F^{\prime \prime}$. Mặt khác, vì $\left|F^{\prime \prime}\right|=k>t$ nên phải có phần tử trong $F^{\prime \prime}$ mà không thuộc vào $F \cap F^{\prime}$. Khi đó, xét $B=F \cup F^{\prime} \cup F^{\prime \prime}$ thì rõ ràng

$\quad\quad\quad\quad\quad |B| \leq\left|F \cup F^{\prime}\right|+\left|F^{\prime \prime}\right| \leq 2 k-t+k<3 k .$

Ta sẽ chứng minh rằng tập hợp này thỏa mãn đề bài. Thật vậy, xét tập hợp $G \in \mathcal{F}$

  • Nếu $G$ là một trong ba tập $F, F^{\prime}, F^{\prime \prime}$ thì có ngay

$\quad\quad\quad\quad\quad\quad\quad\quad\quad\quad |B \cap G| \geq k \geq t+1$

  • Nếu $G \neq F, F^{\prime}, F^{\prime \prime}$ thì nếu $|G \cap F| \geq t+1$ hoặc $\left|G \cap F^{\prime}\right| \geq t+1$ là xong; ngược lại $|G \cap F|=t$ và $t$ phần tử chung này không đồng thời thuộc $F^{\prime \prime}$, nên sẽ còn một phần tử chung khác giữa hai tập hợp $G, F^{\prime^{\prime}}$ do ta có $\left|G \cap F^{\prime \prime}\right| \geq t$, kéo theo $|B \cap G| \geq t+1$.

Trong mọi trường hợp, ta luôn chọn được tập hợp $B \subset X$ thỏa mãn đề bài.

(b) Ta thực hiện xây dựng một họ $\mathcal{F}^{\prime}$ các tập con của $X$ như sau

  • Chọn $t+1$ phần tử nào đó trong tập $B$ ở trên, số cách chọn sẽ nhỏ hơn $C_{3 k}^{t+1}$.
  • Chọn $k-t-1$ phần tử nào đó trong tập $X$ thì có $C_n^{k-t-1}$ cách.

Hợp của hai nhóm này lại thì được một tập con của $X$ có $k$ phần tử (hoặc ít hơn) mà giao với $B$ là $t+1$ phần tử. Từ đó dễ thấy rằng $\mathcal{F} \subset \mathcal{F}^{\prime}$ nên ta phải có $|\mathcal{F}| \leq\left|\mathcal{F}^{\prime}\right|<C_{3 k}^{t+1} C_n^{k-t-1}$. Bài toán được giải quyết.

Nhận xét. Cố định $k, t \geq 2$ và cho $n$ đủ lớn thì ta có

$\quad\quad\quad\quad\quad\quad\quad\quad\quad\quad C_{3 k}^{t+1} C_n^{k-t-1}<C_{n-t}^{k-t} .$

Từ đó, ta có chứng minh của định lý Erdós – Ko – Rado (1938) là: $|\mathcal{F}| \leq C_{n-t}^{k-t}$ với $n$ đủ lớn. Trường hợp $t=1$ thì khi $n \geq 2 k$, ta cũng có $|\mathcal{F}| \leq C_{n-1}^{k-1}$.

Các bài toán theo kiểu family set đã được nghiên cứu rất sâu sắc và vì hầu hết là các đánh giá, ước lượng tổ hợp không có dấu bằng nên đó đều là các kết quả khó.

Bài 8. Cho tam giác $A B C$ nội tiếp trong đường tròn $(O)$ với $B, C$ cố định và $A$ thay đổi trên cung lớn $B C$. Dựng hình bình hành $A B D C$ và $A D$ cắt lại $(B C D)$ ở $K$.

(a) Gọi $R_1, R_2$ lần lượt là bán kính đường tròn ngoại tiếp $(K A B),(K A C)$. Chứng minh rằng tích $R_1 R_2$ không đổi.

(b) Ký hiệu $(T),\left(T^{\prime}\right)$ lần lượt là các đường tròn cùng đi qua $K$, tiếp xúc với $B D$ ở $B$ và tiếp xúc với $C D$ ở $C$. Giả sử $(T),\left(T^{\prime}\right)$ cắt nhau ở $L \neq K$. Chứng minh rằng $A L$ luôn đi qua một điểm cố định.

Lời giải. (a) Gọi $R$ là bán kính của $(O)$, ta có

$\quad\quad\quad\quad\quad\quad\quad\quad \angle A K B=180^{\circ}-\angle B K D=\angle B C D=\angle A B C$

nên theo định lý sin thì

$\quad\quad\quad\quad\quad\quad\quad\quad R_1=\frac{A B}{2 \sin A K B}=\frac{2 R \sin C}{2 \sin B}=R \cdot \frac{\sin C}{\sin B} .$

Tương tự thì $R_2=R \cdot \frac{\sin B}{\sin C}$ nên $R_1 R_2=R^2$, không đổi.

(b) Do các tiếp tuyến nên biến đổi góc được

$\quad\quad\quad\quad\quad\quad \angle K L B=180^{\circ}-\angle K B D, \angle K L C=180^{\circ}-\angle K C D .$

Suy ra

$\quad\quad\quad\quad\quad\quad \angle K L B+\angle K L C=360^{\circ}-(\angle K B D+\angle K C D)=180^{\circ}$

nên $B, L, C$ thẳng hàng. Do $B D$ tiếp xúc với $(K B L)$ nên

$\quad\quad\quad\quad\quad\quad \angle B K L=\angle L B D=\angle C K D,$

mà $K D$ là trung tuyến của tam giác $K B C$ nên $K L$ là đối trung của tam giác này. Gọi $I$ là trung điểm $B C$. Ta có

$\quad\quad\quad\quad\quad\quad \angle K B I=\angle K D C=\angle B A K$

nên $B C$ tiếp xúc với $(A B K)$. Suy ra $\frac{B K}{A B}=\frac{I K}{I A}$. Tương tự thì

$\quad\quad\quad\quad\quad\quad \frac{C K}{A C}=\frac{I K}{I A} \text { nên } \frac{K B}{K C}=\frac{A B}{A C} .$

Theo tính chất quen thuộc của đường đối trung thì

$\quad\quad\quad\quad\quad\quad \frac{L B}{L C}=\frac{K B^2}{K C^2}=\frac{A B^2}{A C^2}$

nên $A L$ cũng là đường đối trung của tam giác $A B C$, diều này cho thấy $A L$ đi qua giao điểm hai tiếp tuyến của $(O)$ ở $B, C$, là điểm cố định.

Nhận xét. Điểm $K$ trong đề bài chính là điểm Humpty của tam giác $A B C$, và cách dựng như trên là một cách tuy dễ nhưng tương đối mới.

 

 

 

 

 

 

 

 

 

 

 

 

 

 

 

 

 

 

 

 

 

 

 

 

 

 

 

 

 

 

 

 

 

 

 

 

 

 

 

 

 

 

 

 

 

 

 

 

 

 

 

 

 

 

 

 

 

 

 

 

 

 

 

 

 

 

 

 

 

 

 

 

 

 

 

 

 

 

 

 

 

 

 

 

 

 

 

 

 

 

 

ĐỀ THI VÀO LỚP 10 CHUYÊN TOÁN PHỔ THÔNG NĂNG KHIẾU 2019

Bài 1. Cho phương trình $a x^{2}+b x+c=0(1)$ thỏa mãn các điều kiện:

$a>0 \text { và } 2 \sqrt{|a c|}<|b|<a+c$

(a) Chứng minh rằng phương trình (1) có hai nghiệm phân biệt $x_{1}, x_{2}$ và

$\left(1-x_{1}\right)\left(1-x_{2}\right)>0 \text { và }\left(1+x_{1}\right)\left(1+x_{2}\right)>0$

(b) Biết rằng $a>c$. Chứng minh rằng $-1<x_{1}, x_{2}<1$

Bài 2. (a) Tìm tất cả những số tự nhiên $n$ sao cho $2^{n}+1$ chia hết cho $9 .$

(b) Cho $n$ là số tự nhiên $n>3$. Chứng minh rằng $2^{n}+1$ không chia hết cho $2^{m}-1$ với mọi số tự nhiên $m$ sao cho $2<m \leq n$.

Bài 3. Cho $a$ và $b$ là hai số thực phân biệt thỏa mãn điều kiện $a^{4}-4 a=b^{4}-4 b$.

(a) Chứng minh rằng $0<a+b<2$.

(b) Biết rằng $a^{4}-4 a=b^{4}-4 b=k>0$. Chứng minh rằng $-\sqrt{k}<a b<0$.

Bài 4. Cho tam giác $A B C$ có $A B<A C$. Gọi $d_{1}$, $d_{2}$ lần lượt là các đường phân giác trong và ngoài góc $\angle B A C$. Gọi $M, N$ lần là hình chiếu vuông góc của $B$ lên $d_{1}, d_{2}$. Gọi $P, Q$ lần lượt là hình chiếu vuông góc của $C$ lên $d_{1}, d_{2}$.

(a) Chứng minh rằng $M N$ và $P Q$ lần lượt đi qua trung điểm của $A B, A C$.

(b) Chứng minh rằng $M N$ và $P Q$ cắt nhau trên $B C$.

(c) Trên $d_{1}$ lấy các điểm $E$ và $F$ sao cho $\angle A B E=\angle B C A$ và $\angle A C F=\angle C B A$. ( $E$ thuộc nữa mặt phẳng bờ $A B$ chứa $C ; F$ thuộc nữa mặt phẳng bờ $A C$ chứa $B)$. Chứng minh rằng $\frac{B E}{C F}=\frac{A B}{A C}$.

(d) Các đường thẳng $B N$ và $C Q$ lần lượt cắt $A C$ và $A B$ tại các điểm $K$ và $L$. Chứng minh rằng các đường thẳng $K E$ và $L F$ cắt nhau trên đường thẳng $B C$.

Bài 5. Trong một buổi gặp gỡ giao lưu giữa các học sinh đến từ $n$ quốc gia, người ta nhận thấy rằng cứ 10 học sinh bất kỳ thì có ít nhất 3 học sinh đến từ cùng một quốc gia.

(a) Gọi $k$ là số các quốc gia có đúng 1 học sinh tham dự buổi gặp gỡ. Chứng minh rằng $n<\frac{k+10}{2}$.

(b) Biết rằng số các học sinh tham dự buổi gặp gỡ là 60 . Chứng minh rằng có thể tìm được ít nhất là 15 học sinh đến cùng một quốc gia.

 

LỜI GIẢI

 

Bài 1. Cho phương trình $a x^{2}+b x+c=0(1)$ thỏa mãn các điều kiện:

$a>0 \text { và } 2 \sqrt{|a c|}<|b|<a+c$

(a) Chứng minh rằng phương trình (1) có hai nghiệm phân biệt $x_{1}, x_{2}$ và

$\left(1-x_{1}\right)\left(1-x_{2}\right)>0 \text { và }\left(1+x_{1}\right)\left(1+x_{2}\right)>0$

(b) Biết rằng $a>c$. Chứng minh rằng $-1<x_{1}, x_{2}<1$

Lời giải.

(a) Có

$|b|>2 \sqrt{|a c|}$

nên $b^{2}>4 a c$. Suy ra $\Delta=b^{2}-4 a c>0$ vậy phương trình có hai nghiệm phân biệt.

$|b|<a+c$

$\Leftrightarrow-a-c<b<a+c $

$\Leftrightarrow\left\{\begin{array}{l}a+b+c>0 \\ a-b+c>0\end{array}\right.$

Suy ra

$\left(1-x_{1}\right)\left(1-x_{2}\right)$

$=1-\left(x_{1}+x_{2}\right)+x_{1} x_{2}$

$=1+\frac{b}{a}+\frac{c}{a}$

$=\frac{a+b+c}{a}>0$

$\left(1+x_{1}\right)\left(1+x_{2}\right)$

$=1+\left(x_{1}+x_{2}\right)+x_{1} x_{2}$

$=1-\frac{b}{a}+\frac{c}{a}$

$=\frac{a-b+c}{a}>0$

(b) Có

$\left(1-x_{1}\right)\left(1-x_{2}\right)>0$

Xét Trường hợp :

$\left\{\begin{array}{l}x_{1}>1 \\ x_{2}>1\end{array} \Rightarrow x_{1} x_{2}>1 \Rightarrow \frac{c}{a}>1 \Rightarrow c>a\right.$

mâu thuẫn với giả thiết $a>c$.

Vậy $x_{1}, x_{2}<1$.

$\left(1+x_{1}\right)\left(1+x_{2}\right)>0$

Xét trường hợp:

$\left\{\begin{array}{l}x_{1}<-1 \\ x_{2}<-1\end{array} \Rightarrow x_{1} x_{2}>1 \Rightarrow \frac{c}{a}>1 \Rightarrow c>a\right.$

mâu thuẫn với giả thiết $a>c$.

Vậy $x_{1}, x_{2}>-1$.

Bài 2. (a) Tìm tất cả những số tự nhiên $n$ sao cho $2^{n}+1$ chia hết cho 9 .

(b) Cho $n$ là số tự nhiên $n>3$. Chứng minh rằng $2^{n}+1$ không chia hết cho $2^{m}-1$ với mọi số tự nhiên $m$ sao cho $2<m \leq n$.

Lời giải.

(a) $n=3 k$, suy ra $2^{n}+1=8^{k}+1 \equiv(-1)^{k}+1(\bmod 9)$. Suy ra $k$ lẻ, $k=$ $2 t+1$. Suy ra $n=3(2 t+1)=6 t+3$.

Nếu $n=3 k+1$ ta có $2^{n}+1=3 \cdot 8^{k}+1 \equiv(-1)^{k} \cdot 3+1(\bmod 9)$, suy ra $2^{n}+1$ không chia hết cho 9 .

Nếu $n=3 k+2$ ta có $2^{n}+1=4 \cdot 8^{k}+1 \equiv 4(-1)^{k}+1$, suy ra $2^{n}+1$ không chia hết cho 9 .

Vậy với $n=6 t+2$, với $t$ là số tự nhiên là các số cần tìm.

(b) Cách 1: Ta có $2^{k m}-1: 2^{m}-1$. Từ $2^{2 n}=\left(2^{n}+1\right)\left(2^{n}-1\right)$ chia hết cho $2^{m}-1$. Đặt $2 n=k m+q(0 \leq q<m)$.

Khi đó $2^{2 n}-1=2^{k m+q}-2^{q}+2^{q}-1=2^{q}\left(2^{k m}-1\right)+2^{q}-1$ chia hết cho $2^{m}-1$, suy ra $2^{q}-1$ chia hết cho $m$ mà $0 \leq 2^{q}-1<2^{m}-1$, suy ra $q=0$. Do đó $2 n=k m$.

Trường hợp 1: Nếu $m$ lẻ, suy ra $k$ chẵn, $k=2 k^{\prime}$, suy ra $n=k^{\prime} m, 2^{n}+1=$ $2^{k^{\prime} m}+1=2^{k^{\prime} m}-1+2$ chia hết cho $2^{m}-1$, suy ra 2 chia hết cho $2^{m}-1$ (vô lý)

Trường hợp 2: Nếu $m$ chẵn $m=2 m^{\prime}$ thì $n=k m^{\prime}$, suy ra $2^{k m^{\prime}}+1$ chia hết cho $2^{m}-1$, mà $2^{m}-1$ chia hết cho $2^{m^{\prime}}-1$ nên $2^{k m^{\prime}}+1$ chia hết cho $2^{m^{\prime}}-1$, suy ra 2 chia hết cho $2^{m^{\prime}}-1$ vô lý vì $m^{\prime}>1$.

Cách 2: Ta có $2^{n-m}\left(2^{m}-1\right): 2^{m}-1$, suy ra $2^{n}-2^{n-m}: 2^{m}-1$, mà $2^{n}+1: 2^{m}-$ 1 suy ra $2^{n-m}+1$ chia hết cho $2^{m}-1$.

Lý luận tương tự ta có $2^{n-k m}+1$ chia hết cho $2^{m}-1$. Giả sử $n=k m+$ $q, 0 \leq q<m$. Chọn $k$ như trên ta có $2^{q}+1$ chia hết cho $2^{m}-1$. Mà $q<m$ nên $2^{q}+1=2^{m}-1$, giải ra $q=1, m=2$ (vô lý).

Bài 3. Cho $a$ và $b$ là hai số thực phân biệt thỏa mãn điều kiện $a^{4}-4 a=$ $b^{4}-4 b$.

(a) Chứng minh rằng $0<a+b<2$.

(b) Biết rằng $a^{4}-4 a=b^{4}-4 b=k>0$. Chứng minh rằng $-\sqrt{k}<a b<0$.

Lời giải.

(a) Ta có $a^{4}-b^{4}=4(a-b)$, mà $a^{4}-b^{4}=(a-b)(a+b)\left(a^{2}+b^{2}\right)$ nên đẳng thức được viết lại thành

$(a-b)(a+b)\left(a^{2}+b^{2}\right)=4(a-b)$

Mà $a \neq b$ nên $(a+b)\left(a^{2}+b^{2}\right)=4$. Vi $a^{2}+b^{2}>0($ do $a, b$ không thể đồng thời bằng 0 ) nên ta có $a+b>0$.

Ngoài ra, ta cũng có đánh giá $a^{2}+b^{2}>\frac{(a+b)^{2}}{2}$ (đẳng thức không xảy ra vì $a \neq b$ ) nên

$4>\frac{(a+b)^{3}}{2} \Leftrightarrow(a+b)^{3}<8 \Leftrightarrow a+b<2 .$

Vậy ta được $0<a+b<2$.

(b) Rõ ràng $a b \neq 0$, ta sẽ chứng minh $a, b$ trái dấu. Ta xét hai trường hợp:

  • Nếu $a>0, b>0$ thì $a^{4}-4 a=a\left(a^{3}-4\right)>0$ nên $a>\sqrt[3]{4}>1$. Tương tự thì $b>1$. Khi đó $a+b>2$, mâu thuẫn với a).

  • Nếu $a<0, b<0$ thì $a+b<0$, cũng mâu thuẫn với a).

Do đó $a, b$ trái dấu và $a b<0$.

Không mất tính tổng quát, giả sử $a<0<b$ thì đặt $c=-a>0$, ta viết lại $c^{4}+4 c=b^{4}-4 b=k>0$. Từ đây dễ thấy $(b-c)\left(b^{2}+c^{2}\right)=4$ và $b \neq c$.

Ta cần chứng minh

$-\sqrt{k}<a b \Leftrightarrow-\sqrt{k}<-b c \Leftrightarrow b c<\sqrt{k} .$

Cộng hai vế của các đẳng thức trên lại, ta có

$2k =b^{4}-4 b+c^{4}+4 c=b^{4}+c^{4}-4(b-c)=b^{4}+c^{4}-(b-c)^{2}\left(b^{2}+c^{2}\right)=2 b c\left(b^{2}-b c+c^{2}\right)$

Suy ra $k=b c\left(b^{2}-b c+c^{2}\right)$, mà $b^{2}-b c+c^{2}>b c$ (đẳng thức không xảy ra vì $b \neq c)$ nên $k>b c \cdot b c=(b c)^{2} \Leftrightarrow b c<\sqrt{k}$. Vậy ta có đpcm.

Bài 4. Cho tam giác $A B C$ có $A B<A C$. Gọi $d_{1}, d_{2}$ lần lượt là các đường phân giác trong và ngoài góc $\angle B A C$. Gọi $M, N$ lần là hình chiếu vuông góc của $B$ lên $d_{1}, d_{2}$. Gọi $P, Q$ lần lượt là hình chiếu vuông góc của $C$ lên $d_{1}, d_{2}$.

(a) Chứng minh rằng $M N$ và $P Q$ lần lượt đi qua trung điểm của $A B, A C$.

(b) Chứng minh rằng $M N$ và $P Q$ cắt nhau trên $B C$.

(c) Trên $d_{1}$ lấy các điểm $E$ và $F$ sao cho $\angle A B E=\angle B C A$ và $\angle A C F=$ $\angle C B A$. ( $E$ thuộc nữa mặt phẳng bờ $A B$ chứa $C ; F$ thuộc nữa mặt phẳng bờ $A C$ chứa $B)$. Chứng minh rằng $\frac{B E}{C F}=\frac{A B}{A C}$.

(d) Các đường thẳng $B N$ và $C Q$ lần lượt cắt $A C$ và $A B$ tại các điểm $K$ và $L$. Chứng minh rằng các đường thẳng $K E$ và $L F$ cắt nhau trên đường thẳng $B C$.

Lời giải.

(a) Tứ giác $A M B N$ có $\angle A=\angle M=\angle N=90^{\circ}$ nên tứ giác $A M B N$ là hình chữ nhật. Suy ra $M N$ đi qua trung điểm $A B$.

Tương tự, $A P C Q$ là hình chữ nhật nên $P Q$ đi qua trung điểm $A C$.

(b) Có: $\angle N M A=\angle B A M=\angle M A C$ nên $M N | A C$ mà theo ý a) $N D$ đi qua trung điểm $A B$ nên ta thu được $N M$ đi qua trung điểm $B C$.

Tương tự, $P Q$ đi qua trung điểm $B C$ nên $M N$ và $P Q$ cắt nhau trên $B C$.

(c) Gọi $T$ là giao điểm của $d_{1}$ và $B C$. Dễ dàng chứng minh được $\triangle A B E \sim$ $A C T(g-g)$ nên $\frac{A B}{A C}=\frac{B E}{C T}$.

Tương tự, $\triangle A B T \sim \triangle A C F(g-g)$ nên $\frac{A B}{A C}=\frac{B T}{C F}$.

Do đó, ta có:

$\left(\frac{A B}{A C}\right)^{2}=\frac{B E \cdot B T}{C T \cdot C F}$

mà $A T$ là phân giác góc $A$ nên

$\frac{B T}{C T}=\frac{A B}{A C}$

Ta thu được

$\frac{A B}{A C}=\frac{B E}{C F}$

(d) $\triangle B E T$ có:

$\angle B E T=\angle E B A+\angle E A B=\angle A C B+\angle C A T=\angle B T E$

nên $\triangle B E T$ cân tại $B$. Suy ra $M$ là trung điểm $E T$.

Có TM $|$ NB nên

$\frac{T M}{N B}=\frac{D M}{D N}=\frac{E M}{K N}$

suy ra $\triangle D M E \sim \triangle D N K(c-g-c)$.

Ta thu được $D, E, K$ thẳng hàng.

Tương tự, $L, D, F$ thẳng hàng ta có điều phải chứng minh.

 

Bài 5. Trong một buổi gặp gỡ giao lưu giữa các học sinh đến từ $n$ quốc gia, người ta nhận thấy rằng cứ 10 học sinh bất kỳ thì có ít nhất 3 học sinh đến từ cùng một quốc gia.

(a) Gọi $k$ là số các quốc gia có đúng 1 học sinh tham dự buổi gặp gỡ. Chứng minh rằng $n<\frac{k+10}{2}$.

(b) Biết rằng số các học sinh tham dự buổi gặp gỡ là 60. Chứng minh rằng có thể tìm được ít nhất là 15 học sinh đến cùng một quốc gia.

Lời giải.

(a) Giả sử ngược lại rằng $n \geq \frac{k+10}{2}$ thì $2 n-k \geq 10$. Gọi $A$ là tập hợp các quốc gia có đúng 1 học sinh tham gia và $B$ là tập hợp các quốc gia còn lại. Khi đó, mỗi quốc gia trong $B$ sẽ có ít nhất 2 học sinh.

Ta chọn tất cả học sinh trong $A$ và mỗi quốc gia trong $B$, chọn 2 học sinh thì có $k+2(n-k)=2 n-k$ học sinh.

Các học sinh này có đặc điểm là: không có 3 học sinh nào đến từ cùng quốc gia. Do $2 n-k \geq 10$ nên có thể chọn ra trong đó 10 học sinh nào đó không thỏa mãn đề bài.

(b) Theo câu a, ta có $2 n-k<10$ nên $2 n-k \leq 9 \Leftrightarrow n \leq \frac{k+9}{2}$.

Do số học sinh tổng cộng là 60 , để chỉ ra có 15 học sinh đến từ cùng quốc gia thì theo nguyên lý Dirichlet, ta chỉ cần chỉ ra rằng

$\frac{60-k}{n-k} \geq 15 \Leftrightarrow 15 n-14 k \leq 60$

Ta sẽ chứng minh đánh giá trên đúng với mọi $(n, k)$. Vì ta đã có $n \leq \frac{k+9}{2}$ nên ta sẽ đưa về chứng $\operatorname{minh} 15\left(\frac{k+9}{2}\right)-14 k \leq 60 \Leftrightarrow k \geq \frac{15}{13}$. Do đó, với $k \geq 2$ thì khẳng định đúng. Tiếp theo, ta xét hai trường hợp

  • Nếu $k=0$ thì theo $(*)$, ta phải có $n \leq 4$ nên $15 n-14 k=15 n \leq 60$, đúng.

  • Nếu $k=1$ thì theo $(*)$, khi đó loại trừ học sinh ở nước đó ra thì còn lại 59 học sinh, đến từ 4 quốc gia. Theo nguyên lý Dirichlet, tồn tại 15 học sinh đến từ cùng quốc gia.

 

 

 

 

 

 

 

 

 

 

 

 

 

 

 

 

 

 

 

 

 

 

 

 

 

 

 

 

 

 

 

 

 

 

 

 

 

 

Đề thi và đáp án tuyển sinh vào 10 TPHCM 2019

Đề thi vào lớp 10 TPHCM năm 2019

Bài 1. Cho parabol $(P): y= -\dfrac{1}{2} x^2 $ và đường thẳng $(d): y= x-4$.

a) Vẽ $(P)$ và $(d)$ trên cùng hệ trục tọa độ.

b) Tìm tọa độ giao điểm của $(P)$ và $(d)$ bằng phép tính.

Giải

a) Bảng giá trị:

  • $y=x-4$

  • $y=-\dfrac{1}{2}x^2$

(Học sinh tự vẽ)

b) Phương trình hoành độ giao điểm của $(P)$ và $(d)$:

$-\dfrac{1}{2}x^2 = x-4 \Leftrightarrow x^2 +2x-8=0 \Leftrightarrow (x+4)(x-2)=0 \Leftrightarrow \left[ \begin{array}{l} x=-4 \Rightarrow y=-8 \\ x=2 \Rightarrow y=-2 \end{array} \right. $

Vậy tọa độ giao điểm của $(P)$ và $(d)$ là $(-4;-8)$ và $(2;-2)$.

Bài 2. Cho phương trình: $2x^2 -3x-1 =0$ có 2 nghiệm là $x_1$, $x_2$.

Không giải phương trình, hãy tính giá trị của biểu thức: $A=\dfrac{x_1-1}{x_2+1} + \dfrac{x_2-1}{x_1+1}$

Giải

Ta có: $\Delta = 9+8=17 >0$

$\Rightarrow $ Phương trình đã cho luôn có hai nghiệm phân biệt $x_1$, $x_2$.

Theo định lý Viete, ta có: $\left\{ \begin{array}{l} x_1 + x_2 = \dfrac{3}{2} \\ x_1x_2 = – \dfrac{1}{2} \end{array} \right. $

$A= \dfrac{x_1-1}{x_2+1} + \dfrac{x_2 -1}{x_1 +1}$

$= \dfrac{x_1^2 -1 + x_2^2-1}{x_1x_2 +x_1+x_2+1} $

$= \dfrac{\left( x_1 + x_2 \right) ^2 – 2x_1x_2 -2}{-\dfrac{1}{2}+ \dfrac{3}{2}+ 1} $

$= \dfrac{\dfrac{9}{4}+1-2}{2} = \dfrac{5}{8} $

Bài 3. Quy tắc sau đây cho ta biết được ngày $n$, tháng $t$, năm $2019$ là ngày thứ mấy trong tuần. Đầu tiên, ta tính giá trị của biểu thức $T=n+ H$, ở đây $H$ được xác định bởi bảng sau:

Sau đó, lấy $T$ chia cho $7$ ta được số dư $r$ ($0 \le r \le 6$)

Nếu $r=0$ thì ngày đó là ngày thứ Bảy.

Nếu $r=1$ thì ngày đó là ngày Chủ Nhật.

Nếu $r=2$ thì ngày đó là ngày thứ Hai.

Nếu $r=6$ thì ngày đó là ngày thứ Sáu.

Ví dụ: Ngày $31/12/2019$ có $n=31$; $t=12$; $H=0 \Rightarrow T=31+0=31$; số $31$ chia cho $7$ có số dư là $3$, nên ngày đó là thứ Ba.

a) Em hãy sử dụng quy tắc trên để xác định các ngày $02/09/2019$ và $20/11/2019$ là thứ mấy?

b) Bạn Hằng tổ chức sinh nhật của mình trong tháng $10/2019$. Hỏi sinh nhật của bạn Hằng là ngày mấy? Biết rằng ngày sinh nhật của Hằng là một bội số của $3$ và là thứ Hai.

Giải

a)

  • Ngày $02/09/2019$ có $n=2$, $t=9$; $H= 0$ suy ra $T= 2+0= 2$; 2 chia $7$ dư $2$ nên đó là ngày thứ Hai.
  • Ngày $20/11/2019$ có $n=20$, $t= 11$, $H=-2$ suy ra $T= 20 -2 =18$; $18$ chia $7$ dư $4$ nên đó là ngày thứ Tư.

b) $t=10$; $H=2$ suy ra $T= n+2$

Vì sinh nhật Hằng là thứ Hai nên $T$ chia $7$ dư $2$, suy ra $n$ chia hết cho $7$

Suy ra $n \in \left\{ 7;14;21;28 \right\} $

Lại có $n$ chia hết cho $3$ nên $n=21$

Vậy sinh nhật của Hằng là $21/10/2019$.

Bài 4. Tại bề mặt đại dương, áp suất nước bằng áp suất khi quyển và là $1$ atm (atmosphere). Bên dưới mặt nước, áp suất nước tăng thêm $1$ atm cho mỗi $10$ mét sâu xuống. Biết rằng mối liên hệ giữa áp suất $y$ (atm) và độ sâu $x$ (m) dưới mặt nước là một hàm số bậc nhất có dạng $y=ax+b$

a) Xác định các hệ số $a$ và $b$

b) Một người thợ lặn đang ở độ sâu bao nhiêu nếu người ấy chịu một áp suất là $2,85$ atm?

Giải

a) Ta có: $y= 1+ \dfrac{x}{10}$

Vậy $a= \dfrac{1}{10}$ và $b=1$

b) Ta có: $2,85= 1+ \dfrac{x}{10} \Rightarrow x= 18,5$ (m)

Vậy người thợ lặn ở độ sâu $18,5$ mét.

Bài 5. Một nhóm gồm $31$ bạn học sinh tổ chức một chuyến đi du lịch (chi phí chuyển đi được chia đều cho mỗi bạn tham gia). Sau khi đã hợp đồng xong, vào giờ chót có $3$ ban bận việc đột xuất không đi được nên họ không đóng tiền. Cả nhóm thống nhất mỗi bạn còn lại sẽ đóng thêm $18 000$ đồng so với dự kiến ban đầu để bù lại cho $3$ bạn không tham gia. Hỏi tổng chi phí chuyến đi là bao nhiêu?

Giải

Tổng số tiền $28$ bạn còn lại đã đóng thêm:

$$ 18000 \cdot 28 = 504000 \text{ (đồng)}$$

Số tiền trên chính là tổng số tiền $3$ bạn phải đóng lúc đầu nếu vẫn đi du lịch, nên số tiền mỗi bạn phải đóng lúc đầu nếu đi đủ $31$ bạn là:

$$ 504000 : 3= 168000 \text{ (đồng)}$$

Tổng chi phí chuyến đi là:

$$ 168000 \cdot 31 = 5208000 \text{ (đồng)}$$

Bài 6. Cuối năm học, các bạn lớp $9A$ chia làm hai nhóm, mỗi nhóm chọn một khu vườn sinh thái ở Bắc bán cầu để tham quan. Khi mở hệ thống định vị GPS, họ phát hiện một sự trùng hợp khá thú vị là hai vị trí mà nhóm chọn đều nằm trên cùng một kinh tuyến và lần lượt ở các vĩ tuyến $47^\circ $ và $72^\circ $.

a) Tính khoảng cách (làm tròn đến hàng trăm) giữa hai vị trí đó, biết rằng kinh tuyến là một cung tròn nối liền hai cực của trái đất và có độ dài khoảng $20 000$ km.

b) Tính (làm tròn đến hàng trăm) độ dài bán kính và đường xích đao của trái đất. Từ kết quả của bán kính (đã làm tròn), hãy tính thể tích của trái đất, biết rằng trái đất có dạng hình cầu và thể tích của hình cầu được tính theo công thức $V= \dfrac{4}{3} \cdot 3,14 \cdot R^3$ với $R$ là bán kính hình cầu.

Giải

a) Ta có: $\angle AOB = 72^\circ – 47^\circ = 25^\circ $

Khoảng cách giữa hai vị trí tham quan: $20000 \cdot \dfrac{25}{180} \approx 2800$ (km)

b) Gọi $R$ là bán kính trái đất.

Độ dài đường xích đạo bằng $2$ lần độ dài đường kinh tuyến và bằng: $40000$ km

$\Rightarrow 2R \cdot 3,14 = 40000 \Rightarrow R \approx 6400$ (km)

Thể tích trái đất: $V = \dfrac{4}{3} \cdot 3,14 \cdot R^3 = 1097509546667 \; (km^3)$

Bài 7. Bạn Dũng trung bình tiêu thụ $15$ ca-lo cho mỗi phút bơi và $10$ ca-lo cho mỗi phút chạy bộ. Hôm nay, Dũng mất $1,5$ giờ cho cả hai hoạt động trên và tiêu thụ hết $1200$ ca-lo. Hỏi hôm nay bạn Dũng mất bao nhiêu thời gian cho mỗi hoạt động?

Giải

$1,5$ giờ $= 90$ phút

Gọi $x$, $y$ (phút) lần lượt là thời gian Dũng mất cho việc bơi và chạy bộ. ($x,y>0$)

Ta có hệ phương trình:

$\left\{ \begin{array}{l} x+ y =90 \\ 15x+ 10y= 1200 \end{array} \right. \Leftrightarrow \left\{ \begin{array}{l} x= 60 \\ y=30 \end{array} \right. $

Vậy Dũng đã bơi $60$ phút và chạy bộ $30$ phút.

Bài 8. Cho tam giác nhọn $ABC$ ($AB<AC$) nội tiếp đường tròn $(O)$. Hai đường cao $BD$ và $CE$ của tam giác $ABC$ cắt nhau tại $H$. Đường thẳng $AH$ cắt $BC$ và $(O)$ lần lượt tại $F$ và $K$ ($K\ne A$). Gọi $L$ là hình chiếu của $D$ lên $AB$.

a) Chứng minh rằng tứ giác $BEDC$ nội tiếp và $BD^2 = BL \cdot BA$

b) Gọi $J$ là giao điểm của $KD$ và $(O)$ ($J \ne K$). Chứng minh $\angle BJK = \angle BDE$

c) Gọi $I$ là giao điểm của $BJ$ và $ED$. Chứng minh tứ giác $ALIJ$ nội tiếp và $I$ là trung điểm của $ED$.

Giải

a) Tứ giác $BEDC$ có $\angle BEC= \angle BDC = 90^\circ$ nên tứ giác $BEDC$ nội tiếp đường tròn đường kính $BC$.

Tam giác $BDA$ vuông tại $D$ có $DL \bot BA$ nên ta có $BD^2=BL \cdot BA$

b) Có $ \angle BJK = \angle BCK =\angle BAK$ mà tứ giác $ADHE$ nội tiếp đường tròn đường kính $AH$ nên $\angle EAH= \angle BDE$ suy ra $\angle BJK =\angle BDE$.

c) Có $\angle BJK=\angle BDE$ suy ra $\Delta BDI \sim \Delta BJD (g-g)$ ta thu được $BD^2=BI \cdot BJ$

mà theo câu a) ta có $BD^2=BL \cdot BA$ nên $\Delta BIL \sim \Delta BAJ (c-g-c)$ suy ra $\angle BLI = \angle BJA$ do đó tứ giác $ALIJ$ nội tiếp.

Có $\angle LEI=\angle ACB=\angle AJB =\angle ELI$ suy ra tam giác $LEI$ cân tại $I$ nên $IL=IE$.

Tương tự $IL=ID$ suy ra $IE=ID (dpcm)$.

 

Đề thi và đáp án tuyển sinh vào lớp 10 PTNK không chuyên 2019

Đề thi vào lớp 10 trường Phổ thông Năng khiếu năm 2019

Bài 1. Tìm $a$ biết

$$\dfrac{\left( \sqrt{a}+1 \right) ^2 – \left( \sqrt{a}-1 \right) ^2}{4\sqrt{a} \left( \sqrt{a}-1 \right) } – \dfrac{\left( \sqrt{2a+1} + \sqrt{a+1} \right) \left( \sqrt{2a+1} – \sqrt{a+1} \right) }{a\left( \sqrt{a}+1 \right) }=1 $$

Giải

Điều kiện: $a>0; a \ne 1$

Ta có:

$\dfrac{\left( \sqrt{a}+1 \right) ^2 – \left( \sqrt{a}-1 \right) ^2}{4\sqrt{a} \left( \sqrt{a}-1 \right) } – \dfrac{\left( \sqrt{2a+1} + \sqrt{a+1} \right) \left( \sqrt{2a+1} – \sqrt{a+1} \right) }{a\left( \sqrt{a}+1 \right) }=1 $

$\Leftrightarrow \dfrac{\left( \sqrt{a}+1 + \sqrt{a} -1 \right) \left( \sqrt{a}+ 1 – \sqrt{a}+1 \right) }{4\sqrt{a}\left( \sqrt{a}-1 \right) } – \dfrac{2a+1 – a-1}{a\left( \sqrt{a}+1 \right) }=1 $

$\Leftrightarrow \dfrac{4\sqrt{a}}{4\sqrt{a}\left( \sqrt{a}-1 \right) }- \dfrac{a}{a\left( \sqrt{a}+ 1\right) } = 1$

$\Leftrightarrow \dfrac{1}{ \sqrt{a}-1 } – \dfrac{1}{\sqrt{a}+ 1}= 1 $

$\Leftrightarrow \dfrac{\sqrt{a}+ 1 – \sqrt{a} +1 }{a-1}=1 $

$\Leftrightarrow \dfrac{2}{a-1}= 1$

$\Leftrightarrow a= 3 $

Vậy $a=3$

Bài 2.

a) Giải phương trình: $\left( \sqrt{x+2}-x \right) \left( \sqrt{2x-5} -1 \right) =0 $

b) Giải hệ phương trình: $\left\{ \begin{array}{l} \sqrt{x+y+3}= \sqrt{2x+3y+1} \\ x(y+1)-4(x+y)+54=0 \end{array} \right. $

Giải

a) Điều kiện: $x \ge \dfrac{5}{2}$

$\left( \sqrt{x+2}-x \right) \left( \sqrt{2x-5} -1 \right) =0 $

$\Leftrightarrow \left[ \begin{array}{l} \sqrt{x+2}= x \\ \sqrt{2x-5} = 1 \end{array} \right.$

$\Leftrightarrow \left[ \begin{array}{l} x+2 = x^2 \\ 2x-5=1 \end{array} \right. $

$\Leftrightarrow \left[ \begin{array}{l} x^2 -x -2 =0 \\ x= 3 \end{array} \right. $

$\Leftrightarrow \left[ \begin{array}{l} x=-1 \text{ (loại)} \\ x= 2 \;\;\; \text{ (loại)} \\ x=3 \;\;\text{ (nhận)} \end{array} \right. $

Vậy $S=\left\{ 3 \right\} $

b) Điều kiện: $\left\{ \begin{array}{l} x+y+3 \ge 0 \\ 2x + 3y +1 \ge 0 \end{array} \right. $

Ta có:

$\left\{ \begin{array}{l} \sqrt{x+y+3}= \sqrt{2x+3y+1} \\ x(y+1)-4(x+y)+54=0 \end{array} \right. $

$\Leftrightarrow \left\{ \begin{array}{l} x+y+3=2x+3y+1 \\ xy+x-4x-4y + 54 =0 \end{array} \right.$

$\Leftrightarrow \left\{ \begin{array}{l} x= -2y +2 \\ (-2y+2)y-3(-2y+2)-4y +54=0 \end{array} \right. $

$\Leftrightarrow \left\{ \begin{array}{l} x= -2y + 2 \\ -2y^2 + 2y + 6y-6 -4y +54=0 \end{array} \right. $

$\Leftrightarrow \left\{ \begin{array}{l} x= -2y+2 \\ -2y^2+4y+48=0 \end{array} \right.$

$\Leftrightarrow \left[ \begin{array}{l} \left\{ \begin{array}{l} x=-10 \\ y=6 \end{array} \right. \;\; \text{ (loại)}\\ \left\{ \begin{array}{l} x=10 \\ y=-4 \end{array} \right. \;\;\; \text{ (nhận)} \end{array} \right. $

Vậy $(x,y)=(10;-4)$

Bài 3. Cho phương trình $x^2-(2m+1)x-12=0 $ $(1)$

a) Với giá trị nào của $m$ thì phương trình (1) có hai nghiệm phân biệt $x_1$, $x_2$ sao cho

$$x_1+x_2 -2x_1x_2=25 $$

b) Tìm $m$ để phương trình $(1)$ có hai nghiệm $x_1$, $x_2$ thỏa:

$$x_1^2 -x_2^2 -7(2m+1)=0 $$

Giải

a) Ta có: $\Delta = (2m+1)^2 + 48 >0$ với mọi $m$

Suy ra phương trình $(1)$ luôn có hai nghiệm phân biệt $x_1$, $x_2$ với mọi $m$

Theo định lý Viete, ta có: $\left\{ \begin{array}{l} S= x_1 + x_2 = 2m+1 \\ P = x_1 x_ 2 = -12 \end{array} \right. $

Ta có: $ x_1 + x_2 -2x_1x_2 =25 \Leftrightarrow 2m+1 +24 =25 \Leftrightarrow m = 0 $

Vậy $m=0$

b) $x_1^2 – x_2^2 – 7(2m+1) =0 $

$\Leftrightarrow \left( x_1-x_2 \right) \left( x_1 + x_2 \right) – 7(2m+1) =0 $

$\Leftrightarrow (2m+1)\left( x_1-x_2 \right) – 7(2m+1) =0 $

$\Leftrightarrow (2m+1)\left( x_1 -x_2 -7 \right) =0 $

$\Leftrightarrow \left[ \begin{array}{l} m= -\dfrac{1}{2} \\ x_1-x_2 =7 \hspace{1cm}(2) \end{array} \right. $

Ta có: $(2)\Leftrightarrow x_1 + x_2 -2 x_2 =7 $

$\Leftrightarrow 2m+1 -2 x_2 = 7\Leftrightarrow x_2 = m-3 $

Mà $x_1 + x_2 = 2m+1$ nên $x_1 = m+4$

Lại có $x_1x_2 =-12  \Rightarrow (m+4)(m-3)=-12 \Leftrightarrow m^2 + m -12 = -12 \Leftrightarrow \left[ \begin{array}{l} m= 0 \\ m=-1 \end{array} \right. $

Vậy $m \in \left\{ -1; -\dfrac{1}{2} ; 0 \right\} $

Bài 4.

a) Từ ngày $1/1/2019$ đến $20/5/2019$, giá bán lẻ xăng RON $95$ có đúng bốn lần tăng và một lần giảm. Các thời điểm thay đổi giá xăng RON $95$ trong năm $2019$ (tính đến ngày $20/5/2019$) được cho bởi bảng sau:

Từ $16$ giờ chiều $2/5/2019$ giá bán lẻ $1$ lít xăng RON $95$ tăng thêm khoảng $25\%$ so với giá $1$ lít xăng RON $95$ ngày $1/1/2019$. Nếu ông $A$ mua $100$ lít xăng RON $95$ ngày $2/1/2019$ thì cũng với số tiền đó ông $A$ sẽ mua được bao nhiêu lít xăng RON $95$ vào ngày $3/5/2019$? Cũng trong $2$ ngày đó ($2/1$ và $3/5$), ông $B$ đã mua tổng cộng $200$ lít xăng RON $95$ với tổng số tiền $3850000$ đồng, hỏi ông $B$ đã mua bao nhiêu lít xăng vào ngày $3/5/2019$?

b) Tứ giác $ABCD$ có chu vi $18 \; cm$, $AB=\dfrac{3}{4} BC$, $CD= \dfrac{5}{4}BC$ và $AD=2AB$.

Tính độ dài các cạnh của tứ giác $ABCD$. Biết $AC=CD$, tính diện tích tứ giác $ABCD$.

Giải

a)  Giá $1$ lít xăng RON $95$ vào $16$ giờ chiều $2/5/2019$ là: $$ 17600 \left( 1+ 25\% \right) = 22000 \; \text{(đồng)} $$

Số tiền ông $A$ đã dùng để mua $100$ lít xăng vào ngày $2/1/2019$ là: $$ 100 \cdot 17600 = 1760000 \; \text{(đồng)} $$

Lượng xăng ông $A$ có thể mua được vào ngày $3/5/2019$ với số tiền trên là: $$ 1760000 : 22000 = 80 \; (l)$$

Gọi $x$, $y$ (lít) lần lượt là lương xăng ông $B$ đã mua vào ngày $2/1$ và $3/5$. ($x, y >0$)

Ta có hệ sau:

$\left\{ \begin{array}{l} x+ y= 200 \\ 17600x + 22000y = 3850000 \end{array} \right. \Leftrightarrow \left\{ \begin{array}{l} x= 125 \\ y= 75 \end{array} \right. $

Vậy ông $B$ đã mua $75$ lít xăng RON $95$ vào ngày $3/5/2019$.

b) Ta có:

$AB+BC+CD+AD= 18 $

$\Leftrightarrow \dfrac{3}{4}BC + BC + \dfrac{5}{4}BC + 2 \cdot \dfrac{3}{4}BC = 18 $

$\Leftrightarrow \dfrac{9}{2}BC = 18 \Leftrightarrow BC= 4 \; cm $

$\Rightarrow AB = 3 \; cm; CD= 5 \; cm; AD = 6 \; cm$

Tam giác $ABC$ có $AB^2+BC^2= 3^2 + 4^2 = 25 = AC^2$ nên tam giác $ABC$ vuông tại $B$

$\Rightarrow S_{ABC}= \dfrac{1}{2} \cdot AB\cdot BC = \dfrac{1}{2} \cdot 3\cdot 4= 6 \, cm^2$

Tam giác $ACD$ có $AC=CD$ nên cân tại $C$.

Gọi $M$ là trung điểm của $AD$, suy ra $AM=MD=\dfrac{AD}{2}=3 \; cm$ và $CM \bot AD$

Tam giác $ACM$ vuông tại $M$ nên

$CM^2 + AM^2 = AC^2 \Rightarrow CM^2 = 5^2-3^2 = 16 \Rightarrow CM = 4\; cm$

$\Rightarrow S_{ACD}= \dfrac{1}{2} \cdot CM \cdot AD = \dfrac{1}{2} \cdot 4 \cdot 6 = 12 \, cm^2$

Vậy $S_{ABCD}=S_{ABC}+S_{ACD} = 6+12 =18 \; cm^2$

Bài 5. Hình chữ nhật $ABCD$ nội tiếp đường tròn $(T)$ có tâm $O$, bán kính $R=2a$. Tiếp tuyến của $(T)$ tại $C$ cắt các tia $AB$, $AD$ lần lượt tại $E$, $F$.

a) Chứng minh $AB\cdot AE = AD \cdot AF$ và $BEFD$ là tứ giác nội tiếp.

b) Đường thẳng $d$ đi qua $A$, $d$ vuông góc với $BD$ và $d$ cắt $(T)$, $EF$ theo thứ tự tại $M$, $N$ ($M \ne A$). Chứng minh $BMNE$ là tứ giác nội tiếp và $N$ là trung điểm của $EF$.

c) Gọi $I$ là tâm đường tròn ngoại tiếp tam giác $BEF$. Tính IN theo $a$

Giải

a) $ABCD$ là hình chữ nhật nội tiếp đường tròn $(O)$ nên $O$ là trung điểm của $AC, BD$.

Ta có $OC \bot EF$ (do EF là tiếp tuyến), $AD \bot CD$ do $ABCD$ là hình chữ nhật.

Tam giác $ACF$ vuông tại $C$ có $CD$ là đường cao nên $AD \cdot AF = AC^2$.

Tương tự, tam giác $ACE$ vuông tại $C$ có $CB$ là đường cao nên $AB \cdot AE = AC^2$.

Do đó $AD \cdot AF = AB \cdot AE$.

Suy ra $\dfrac{AD}{AE} = \dfrac{AB}{AF}$.

Xét tam giác $ABD$ và $AFE$ có $\angle A$ chung và $\dfrac{AD}{AE} = \dfrac{AB}{AF}$

nên $\triangle ABD \backsim \triangle AFE$, suy ra $\angle ABD = \angle AFE$, suy ra tứ giác $BDFE$ nội tiếp.

(Cách khác: $\angle ABD = \angle ACD$ mà $\angle ACD = \angle AFE$ (Cùng phụ $\angle DCF$)

Suy ra $\angle ABD = \angle AFE$).

b) Ta có $\angle AMB = \angle ACB$ (cùng chắn cung AB), mà $\angle ACB = \angle BEN$

(cùng phụ $\angle BCE$)

Suy ra $\angle AMB = \angle BEN$, suy ra $BENM$ nội tiếp.

Ta có $\angle BMA = \angle BDA$ (cùng chắn cung $AB$), mà $\angle BDA = \angle BAM$ (cùng phụ với $\angle ABD$)

Suy ra $\angle BMA = \angle BAM = \angle NAE$. Vậy $\angle NEA = \angle NAE$.

Tam giác $NAE$ có $\angle NEA = \angle NAE$ nên cân tại $N$ hay $NA = NE$.

Mà $\angle NEA + \angle NFA = 90^\circ = \angle NAE + \angle NAF$, suy ra $\angle NFA = \angle NAF$, suy ra $NA = NF$.

Vậy $NE = NA = NF$ hay $N$ là trung điểm $EF$.

c)  Ta có $N$ là trung điểm $EF$ nên $IN \bot EF$, mà $AO \bot EF$, suy ra $IN \parallel AO$;

Và $IO \bot BD, AN \bot BD$, suy ra $IO \parallel AN$;

Do đó tứ giác $ANIO$ là hình bình hành, suy ra $IN = AO = R$.

Đề và lời giải thi chọn đội dự tuyển năm học 2018-2019

Bài 1. Tìm tất cả các hàm số $f:\mathbb R\rightarrow \mathbb R$ thoả mãn:
i) $f(-x)=-f(x)\ \forall x\in \mathbb R$.
ii) $f(f(x)-y)=2x+f(f(y+x))\ \forall x,y\in \mathbb R$.

Bài 2. Tìm tất cả các bộ số tự nhiên $(a,b,c)$ để $a^2+2b+c,b^2+2c+a,c^2+2a+b$ đều là các số chính phương.

Bài 3. Cho tập hợp $X={1,2,\ldots,396}$. Gọi $S_1,S_2,\ldots,S_k$ là $k$ tập con khác nhau của $X$ thoả mãn đồng thời hai điều kiện sau:

i)$|S_1|=|S_2|=\ldots=|S_k|=198$.
ii) $|S_i\cap S_j|\le 99\ \forall i,j\in \mathbb N^*, 1\le i<j\le k$.

Chứng minh rằng $k\le 6^{50}$.

Bài 4. Cho tam giác $ABC$ nhọn. Đường tròn thay đổi qua $B,C$ cắt các cạnh $AB,AC$ lần lượt tại $D,E$.

a) Gọi $H,K$ lần lượt là hình chiếu của $B$ trên $CD$ và $DE$. Chứng minh rằng $HK$ luôn đi qua một điểm cố định.
b) Gọi $Q$ là hình chiếu của $C$ trên $DE$. Đường tròn ngoại tiếp tam giác $BDK$ cắt $BC$ tại $M$, đường tròn ngoại tiếp tam giác $CEQ$ cắt $BC$ tại $N$. $KM,QN$ cắt nhau tại $X$. Chứng minh rằng $X$ thuộc một đường thẳng cố định.

Lời giải

Bài 1.

Trong điều kiện $(ii),$ thay $x$ bởi $-x,$ ta được
$$
f(f(-x)-y)=-2x+f(f(y)-x)),
$$
hay
$$-f(f(x)+y)=-2x+f(f(y)-x),\text{ với mọi } x,y\in\mathbb{R}. (*) $$

Thay vai trò của $x$ và $y$ trong $(ii)$, ta có
$
f(f(y)-x)=2y+f(f(x)+y),\text{ với mọi }x,y\in\mathbb{R}.
$
Thay vào $(*)$, ta có
$$
-f(f(x)+y)=-2x+2y+f(f(x)+y),
$$
hay
$$
f(f(x)+y)=x-y,\text{ với mọi }x,y\in\mathbb{R}.
$$
Thay $y$ bởi $f(y),$ ta có
$$
f(f(x)+f(y))=x-f(y),\text{ với mọi }x,y\in\mathbb{R}.
$$
Đổi vai trò của $x,y$, ta thu được
$$
x-f(y)=y-f(x), \text{ tức là } f(x)=-x+c,\text{ với mọi }x\in\mathbb{R}.
$$
Thay vào đề bài, ta suy ra $c=0.$ Vậy hàm số cần tìm là $f(x)=-x.$

Bài 2.

Không mất tính tổng quát, ta giả sử $a=\min (a,b,c).$ Nếu $a=0$ thì ta có $2b+c,b^2+2c,c^2+b$ đều là các số chính phương.
Nếu như $b \le c$ thì $c^2 \le c^2+b \le c^2+c <(c+1)^2$ là số chính phương, kéo theo $c^2+b=c^2$ nên $b=0$. Từ đây dễ dàng có $c=0$. Tương tự nếu $c \le b$ cũng có $b=c=0$. \medskip

Do đó, trong trường hợp này, ta có bộ nghiệm $(a,b,c)=(0,0,0)$.
Ta xét các trường hợp sau ứng với $a>0.$

  • Nếu $a\le b\le c.$ Khi đó $c^2< c^2+2a+b\le c^2+3c<(c+2)^2$. Do đó $c^2+2a+b=(c+1)^2,$ hay $2a+b=2c+1.$ Ta cũng có
    $$
    b^2 < b^2+2c+a = b^2+2a+b-1+a \leq b^2+4b – 1 < (b+2)^2,
    $$
    tức là
    $$
    b^2+2c+a=(b+1)^2,\ 2c+a=2b+1.
    $$
    Đẳng thức xảy ra khi $a=1,b=c$, từ đây dễ dàng tìm được $a=b=c=1$. Thử lại ta thấy bộ số này thỏa mãn.
  • Nếu $a \leq c \leq b$. Khi đó $b^2 < b^2+2c+a \leq b^2 + 3b < (b+2)^2$, tức $b^2+2c+a=(b+1)^2$ và $2c+a=2b+1$. Ta suy ra
    $$4a+2b = 4a+2c+a-1 \leq 7c – 1 < 8c+8.
    $$
    Do đó $2a+b < 4c+4$ và $c^2 < c^2+2a+b < (c+2)^2$. Do đó $2a+b=2c+1$. Kết hợp với $2c+a=2b+1$, ta suy ra
    $$
    (a,b,c)=(a,3a-2,\frac{5a-3}{2}).
    $$
    Do đó $a$ lẻ và trường hợp $a=1$ đã xét nên ở đây ta đặt $a=2t+1$, với $t \geq 1$. Khi đó $(a,b,c)=(2t+1,6t+1,5t+1)$. Vì $b^2+2c+a$ và $c^2+2a+b$ là các số chính phương nên ta xét điều kiện để $a^2+2b+c=4t^2+21t+4$ là số chính phương. Với $t \geq 3$, ta có
    $$
    (2t+4)^2 < 4t^2+21t+4<(2t+6)^2
    $$

Do đó $4t^2+21t+4 = (2t+5)^2$ và $t=21$. Như vậy $t \in {1,2,21}$. Thử trực tiếp, ta thấy chỉ có $t=21$ là thỏa mãn ứng với $(a,b,c)=(43,127,106)$.

Vậy tất cả bộ ba số thỏa mãn đề bài là $$(a,b,c)=(0,0,0),(1,1,1),(43,127,106).$$

Bài 3.

Vì $|S_i\cap S_j|\le 99$ với mọi $1\le i<j\le k$ nên mỗi bộ $100$ phần tử chỉ có thể được chứa tối đa trong $1$ tập hợp. Ta đếm các bộ $\{x_1,x_2,\ldots,x_{100},M \}$, trong đó $x_i\in X$ với mọi $i$ và $M$ là một trong các tập $S_i$, $M$ chứa $x_1,x_2,\ldots,x_{100}$.

  • Số cách chọn tập $M$ là $k$. Số cách chọn $100$ phần tử trong $M$ là $C^{100}_{198}.$
  • Số cách chọn $x_1,x_2,\ldots,x_{100}$ từ $X$ là $C^{100}_{396}.$ Với mỗi bộ $100$ phần tử như vậy, có tối đa $1$ tập $S_i$ thỏa mãn $S_i$ chứa $x_1,x_2,\ldots,x_{100}.$

Do đó ta có bất đẳng thức
$ kC^{100}{198} \le C^{100}{396} $
hay

$k \le \dfrac{C^{100}{396}}{C^{100}{198}}$

$=\dfrac{396!100!98!}{100!296!198!}$

$=\dfrac{396!98!}{198!296!} $

$=\dfrac{297\cdot 298 \ldots 396}{99\cdot 100\ldots 198}$
$=\dfrac{297\cdot 299\ldots 395}{99\cdot 100\ldots 148}\cdot\dfrac{298}{149}\cdot \dfrac{300}{150}\ldots \dfrac{396}{198}$
$\le 3^{50}\cdot 2^{50}=6^{50}.$

Ta có đpcm.

Bài 4.

(a) Gọi $F$ là giao điểm của $KH$ và $AC.$ Ta chứng minh $F$ cố định. Ta có tứ giác $BDEC$ nội tiếp nên $\angle BDC=\angle BEC.$ Tứ giác $KDHB$ cũng nội tiếp nên ta suy ra $\angle BDC=\angle BKF.$ \medskip

Do đó $\angle BEC=\angle BKF,$ tức là tứ giác $KEFB$ nội tiếp. Khi đó ta có $$\angle EFB=180^{\circ}-\angle BKE =90^{\circ}.$$ Do đó $BF\perp AC,$ tức là điểm $F$ cố định.

(b) Tứ giác $DKMB$ nội tiếp nên $\angle BMK=\angle KDB$. Ta suy ra
$$\angle NMX=\angle EDA.$$
Ta có $EQCN$ nội tiếp nên $$\angle QNC=\angle QEC, \text{ hay } \angle MNX=\angle AED.$$
Từ đó, ta suy ra $\triangle MNX\sim \triangle DEA.$ Gọi $G$ là chân đường cao từ $A$ đến $BC$ và $AG$ cắt $DE$ tại $P$. Khi đó $BC\perp AG$. Mà $BC\perp DM$ và $BC\perp EN$ nên
$$
AC \parallel DM \parallel EN.
$$

Do đó $\dfrac{DP}{PE}=\dfrac{MG}{GN}$. Mà $\triangle ADE\sim \triangle XMN$ nên $\angle XMN=\angle EPA.$ Mà $$\angle EPA=180^{\circ}-\angle PAC-\angle PEA$$ nên ta có
$$
\angle EPA=180^{\circ}-(90^{\circ}-\angle C)-B={\rm const}.
$$
Do đó $\angle XGN$ không đổi. Mà $G$ là điểm cố định nên $GX$ cố định. Như vậy $X$ di chuyển trên đường cố định.

 

Đề thi và đáp án vào lớp 10 TPHCM 2017

I. ĐỀ

Câu 1.
a) Giải các phương trình: $x^2=(x-1)(3x-2)$.
b) Một miếng đất hình chữ nhật có chu vi $100m$. Tính chiều dài và chiều rộng của miếng đất biết rằng 5 lần chiều rộng hơn 2 lần chiều dài $40m$.

Câu 2. Trong mặt phẳng tọa độ $Oxy$:
a) Vẽ đồ thị $(P)$ của hàm số $y=\dfrac{1}{4}x^2$.
b) Cho đường thẳng $(D):y=\dfrac{3}{2}x+m$ đi qua điểm $C(6;7)$. Tìm tọa độ giao điểm $(D)$ và $(P)$.
Câu 3.
a) Thu gọn biểu thức $A=(\sqrt{3}+1)\sqrt{\dfrac{14-6\sqrt{3}}{5+\sqrt{3}}}$.
b) Lúc 6 giờ sáng , bạn An đi xe đạp từ nhà (điểm $A$) đến trường (điểm $B$) phải leo lên và xuống một con dốc (như hình bên dưới). Cho biết đoạn thằng $AB$ dài $762m$, góc $A=6^\circ$, góc $B=4^\circ$.

  1. Tính chiều cao $h$ của con dốc.
  2. Hỏi bạn An đến trường lúc mấy giờ? Biết rằng tốc độ trung bình lên dốc là $4km/h$ và tốc độ trung bình xuống dốc là $19km/h$.

Câu 4. Cho phương trình: $x^2-(2m-1)x+m^2-1=0\,(1)$ ($x$ là ẩn số).

a) Tìm điều kiện của $m$ để phương trình $(1)$ có 2 nghiệm phân biệt.
b) Định $m$ để hai nghiệm $x_1$, $x_2$ của phương trình $(1)$ thỏa mãn:
$$(x_1-x_2)^2=x_1-3x_2$$
Câu 5. Cho tam giác $ABC$ vuông tại $A$. Đường tròn tâm $O$ đường kính $AB$ cắt các đoạn $BC$ và $OC$ lần lượt tại $D$ và $I$. Gọi $H$ là hình chiếu của $A$ lên $OC$; $AH$ cắt $BC$ tại $M$.
a) Chứng minh tứ giác $ACDH$ nội tiếp và $\angle{CHD}=\angle{ABC}$.
b) Chứng minh hai tam giác $OHB$ và $OBC$ đồng dạng và $HM$ là tia phân giác của góc $BHD$.
c) Gọi $K$ là trung điểm $BD$. Chứng minh $MD.BC=MB.CD$ và $MB\cdot MD=MK\cdot MC$.
d) Gọi $E$ là giao điểm của $AM$ và $OK$; $J$ là giao điểm của $IM$ và $(O)$ ($J$ khác $I$). Chứng minh hai đường thẳng $OC$ và $EJ$ cắt nhau tại một điểm nằm trên $(O)$.

II. ĐÁP ÁN

Câu 1.
a) $x^2 = (x-1)(3x-2) $
$\Leftrightarrow x^2= 3x^2 – 5x + 2 $
$\Leftrightarrow 2x^2 – 5x+2=0 $
$\Leftrightarrow 2x^2 – 4x -x +2 =0 $
$\Leftrightarrow 2x(x-2)-(x-2) =0 $
$\Leftrightarrow (x-2)\left( 2x-1 \right) =0 $

$\Leftrightarrow  x=2$ hoặc $x=\dfrac{1}{2} $
b) Gọi $a$, $b$ (m) lần lượt là chiều dài và chiều rộng của hình chữ nhật. ($a,b >0$)
Ta có hệ phương trình:
$2(a+b) = 100$ và  $5b-2a=40$
$\Leftrightarrow a=30$ và $b= 20$
Vậy chiều dài và chiều rộng của hình chữ nhật lần lượt là 30m và 20m.

Câu 2. Trong mặt phẳng tọa độ $Oxy$:
a) Đồ thị:

Đồ thị $(P)$ đi qua điểm $(2; 1)$, $(-2;1)$ và $O(0;0)$
b) Đường thẳng $(D)$ đi qua điểm $C(6;7)$ nên
$7=\dfrac{3}{2}.6+m \Rightarrow m= -2$
Do đó phương trình đường thẳng $(D)$ là $(D):y=\dfrac{3}{2}x-2$.
Phương trình hoành độ giao điểm của $(D)$ và $(P)$ là:

$\dfrac{3}{2}x-2= \dfrac{1}{4}x^2 $
$\Leftrightarrow x^2 – 6x+8 =0 $
$\Leftrightarrow x= 4 \Rightarrow y= 4 $ hoặc $x=2 \Rightarrow y= 1$
Vậy các giao điểm của $(D)$ và $(P)$ có tọa độ là $(4;4)$ và $(2,1)$
Câu 3.
a) $\left( \sqrt{3}+1 \right) \sqrt{\dfrac{14-6\sqrt{3}}{5+\sqrt{3}}} = \left( \sqrt{3}+1 \right) \sqrt{\dfrac{20+4\sqrt{3}-10\sqrt{3}-6}{5+\sqrt{3}}} $
$= \left( \sqrt{3}+1 \right) \sqrt{\dfrac{\left( 4-2\sqrt{3}\right) \left( 5+ \sqrt{3} \right) }{5 + \sqrt{3}}} = \left( \sqrt{3}+1 \right) \sqrt{\left( \sqrt{3}-1 \right) ^2} $
$= \left( \sqrt{3}+ 1 \right) \left( \sqrt{3}-1 \right) =3-1 =2$
b)

  1. Ta có:
    $AH = h.cotg \angle CAH= h.cotg \; 6^\circ $
    $BH = h.cotg \angle CBH= h.cotg \; 4^\circ$
    Mà $AH + BH = AB$ nên
    $h.cotg \; 6^\circ + h.cotg \; 4^\circ = 762 $
    $\Leftrightarrow h= \dfrac{762}{cotg \; 6^\circ + cotg \; 4^\circ } $ $\Leftrightarrow h \approx 32$
    Vậy chiều cao của con dốc là $h \approx 32m$
  2.  $AC= \dfrac{h}{\sin \angle CAH} \approx \dfrac{32}{\sin 6^\circ }$
    Vận tốc An lên dốc là $4\; km/h = 4000 \; m /h$
    Thời gian An lên dốc là $\dfrac{\dfrac{32}{\sin 6^\circ }}{4000}$ (giờ)
    $BC= \dfrac{h}{\sin \angle CBH} \approx \dfrac{32}{\sin 4^\circ }$
    Vận tốc An xuống dốc là $19 \; km/h = 19000 \; m/h$
    Thời gian An xuống dốc là $\dfrac{\dfrac{32}{\sin 4^\circ }}{19000}$ (giờ)
    Thời gian để An đến trường là $\dfrac{\dfrac{32}{\sin 6^\circ }}{4000} + \dfrac{\dfrac{32}{\sin 4^\circ }}{19000} \approx 0.1$ (giờ) $\approx 6$ (phút)
    Vậy An đến trường lúc 6 giờ 6 phút.

Câu 4. $x^2 – (2m-1)x + m^2 -1 =0$ (1)

a) Để phương trình (1) có hai nghiệm phân biệt thì
$a=1 \ne 0$ và $\Delta >0 $
$\Leftrightarrow (2m-1)^2 – 4 \left( m^2 -1 \right) >0$
$\Leftrightarrow 4m^2 – 4m +1 – 4m^2 + 4 >0 \Leftrightarrow m < \dfrac{5}{4}$
b) Để phương trình có hai nghiệm $x_1$, $x_2$ thì $a=1 \ne 0$ và $\Delta \ge 0 $ $\Rightarrow m \le \dfrac{5}{4}$
Theo Viet, ta có: $S= 2m-1 $, $P= m^2 -1$
$\left( x_1 -x_2 \right) ^2 = x_1 – 3x_2 $
$\Leftrightarrow \left( x_1 + x_2 \right) ^2 = x_1 + x_2 + 4x_1x_2 -4x_2 $
$\Leftrightarrow (2m-1)^2 = 2m-1 + 4m^2 – 4 – 4x_2 $
$\Leftrightarrow 4m^2 -4m +1 = 2m -1 + 4m^2 -4 – 4x_2 $
$\Leftrightarrow 4x_2 = 6m-6 \Leftrightarrow x_2 = \dfrac{3}{2}m – \dfrac{3}{2}$
$S= x_1 + x_2 = 2m -1 \Rightarrow x_1 = \dfrac{1}{2}m+ \dfrac{1}{2}$
$P = x_1x_2 = m^2 -1 $
$\Rightarrow \left( \dfrac{1}{2}m + \dfrac{1}{2} \right) \left( \dfrac{3}{2}m – \dfrac{3}{2} \right) = m^2 -1 \Leftrightarrow m^2 -1 =0 \Leftrightarrow
m =1 (n)$ hay
m= -1 (n)
Vậy $m=1$ hoặc $m=-1$

Câu 5.

Cho tam giác $ABC$ vuông tại $A$. Đường tròn tâm $O$ đường kính $AB$ cắt các đoạn $BC$ và $OC$ lần lượt tại $D$ và $I$. Gọi $H$ là hình chiếu của $A$ lên $OC$; $AH$ cắt $BC$ tại $M$.
a) $\angle ADB = 90^\circ $ (góc nội tiếp chắn nửa đường tròn)
$\Rightarrow \angle AHC = \angle ADC = 90^\circ \Rightarrow ACDH$ là tứ giác nội tiếp.
$\Rightarrow \angle CAD= \angle CHD$.
Mà $\angle CAD= \angle ABC$ (cùng phụ với $\angle ACB$) nên $\angle CHD = \angle ABC$.
b) Theo câu a), ta có: $\angle CHD = \angle ABC \Rightarrow OBDH$ là tứ giác nội tiếp.
$\Rightarrow \angle OHB = \angle ODB$.
Mà $\angle ODB = \angle OBD$ nên $\angle OHB = \angle OBD \Rightarrow \triangle OHB \backsim \triangle OBC$
$\angle OHB = \angle OBD = \angle CHD \Rightarrow 90^\circ – \angle OHB = 90^\circ – \angle CHD \Rightarrow \angle BHM = \angle DHM$.
Do đó $HM$ là tia phân giác của $\angle BHD$
c) $HM$ là phân giác $\angle BHD$ mà $HM \bot HC$ nên $HC$ là phân giác ngoài của $\angle BHD$.
Do đó ta có $\dfrac{MB}{MD}= \dfrac{HB}{HD}= \dfrac{CB}{CD} \Rightarrow MD.BC= MB.CD$
Tiếp tuyến tại $B$ của $(O)$ cắt $AM$ tại $E$.
$\Rightarrow \angle OBE =90 ^\circ \Rightarrow OBEH$ là tứ giác nội tiếp. $\Rightarrow \angle BOE = \angle BHE$, mà $\angle BHE = \angle DHE$ nên $\angle BOE = \angle DHE$ (1)
Lại có $OBDH$ nội tiếp (cmt) nên 5 điểm $O$, $B$, $E$, $D$, $H$ cùng nằm trên một đường tròn.
$\Rightarrow OHDE$ nội tiếp $\Rightarrow \angle DHE = \angle DOE$ (2)
Từ (1) và (2) suy ra $\angle BOE = \angle DOE \Rightarrow OE$ là phân giác $\angle BOD$.
Do đó $O$, $K$, $E$ thẳng hàng.
$\Rightarrow EK \bot BC $
$\angle EKC = \angle EHC =90^\circ \Rightarrow EKHC$ nội tiếp $\Rightarrow MK.MC = MH.ME$.
$BHDE$ nội tiếp nên $MB.MD = MH.ME$.
Vậy $MB.MD = MK.MC$
d) Gọi $F$ là giao điểm của $EJ$ và $OC$.
Ta có $MH.ME = MB.MD$, $MB.MD = MI.MJ$ nên $MH.ME= MI.MJ \ \Rightarrow \triangle MJE \backsim \triangle MHI \Rightarrow \angle MJE = \angle MHI = 90^\circ \Rightarrow \angle IJF = 90^\circ
\Rightarrow \angle IJF$ là góc nội tiếp chắn nửa đường tròn $(O)$.
Do đó $F$ nằm trên đường tròn $(O)$.
Vậy $EJ$ và $OC$ cắt nhau tại điểm $F$ nằm trên đường tròn.